You are on page 1of 29

INCONTESTABILITY establish such defense by satisfactory and convincing evidence

rests upon the provider or insurer. In any case, with or without the
PHILAMCARE HEALTH SYSTEMS, INC., petitioner, vs. COURT authority to investigate, petitioner is liable for claims made under
OF APPEALS and JULITA TRINOS, respondents. [G.R. No. the contract. Having assumed a responsibility under the
125678. March 18, 2002] YNARES-SANTIAGO, J.: agreement, petitioner is bound to answer the same to the extent
agreed upon. In the end, the liability of the health care provider
Facts: attaches once the member is hospitalized for the disease or injury
Ernani Trinos applied for a health care coverage with petitioner covered by the agreement or whenever he avails of the covered
Philamcare Health Systems, Inc. benefits which he has prepaid.

Under the agreement, respondents husband was entitled to Under the title Claim procedures of expenses in the Policy, the
avail of hospitalization benefits. Upon the termination of the defendant Philamcare Health Systems Inc. had twelve months from
agreement, the same was extended for another year from March 1, the date of issuance of the Agreement within which to contest the
1989 to March 1, 1990, then from March 1, 1990 to June 1, 1990. membership of the patient if he had previous ailment of asthma,
and six months from the issuance of the agreement if the patient
was sick of diabetes or hypertension. The periods having expired,
During the period of his coverage, Ernani suffered a heart the defense of concealment or misrepresentation no longer lie.
attack and was confined at the Manila Medical Center (MMC) for
one month beginning March 9, 1990. While her husband was in the Soliman v. US Life- Rescind Contract of Insurance
hospital, respondent tried to claim the benefits under the health
care agreement. However, petitioner denied her claim saying that 104 PHIL 1046
the Health Care Agreement was void. According to petitioner, there
was a concealment regarding Ernanis medical history. Doctors at
the MMC allegedly discovered at the time of Ernanis confinement Facts:
that he was hypertensive, diabetic and asthmatic, contrary to his
answer in the application form. Thus, respondent paid the > US Life issued a 20 yr endowment life policy on the joint lives of
hospitalization expenses herself. Later, he was admitted at the Patricio Soliman and his wife Rosario, each of them being the
Chinese General Hospital. Due to financial difficulties, however, beneficiary of the other.
respondent brought her husband home again. In the morning of
April 13, 1990, Ernani died. > In Mar. 1949, the spouses were informed that the premium for
Jan 1949 was still unpaid notwithstanding that the 31-day grace
Thereafter, respondent instituted wan action for damages period has already expired, and they were furnished at the same
against petitioner asking for reimbursement of her expenses plus time long-form health certificates for the reinstatement of the
moral damages and attorneys fees. RTC and CA bid favor to policies.
respondent.
> In Apr 1949, they submitted the certificates and paid the
Issue: premiums.
Whether or not inconstestability clause applies?
> In Jan. 1950, Rosario died of acute dilation of the heart, and
thereafter, Patricio filed a claim for the proceeds of the insurance.
Held:
> US life denied the claim and filed for the rescission of the
contract on the ground that the certificates failed to disclose that
Yes. Concealment as a defense for the health care provider or Rosario had been suffering from bronchial asthma for 3 years prior
insurer to avoid liability is an affirmative defense and the duty to to their submission.
Issue: which the defendant duly excepted and filed a motion for a new
trial, which was overruled.
Whether or not the contract can still be rescinded.
Based upon those facts the plaintiff vigorously contended in the
Held: lower court and now contends in the court, that section 47 of the
Insurance Act should be applied, and that when so applied,
Yes. defendant is barred and estopped to plead and set forth the
matters alleged in its special defense. That section is as follows:
The insurer is once again given two years from the date of
reinstatement to investigate into the veracity of the facts Whenever a right to rescind a contract of insurance is given
represented by the insured in the application for reinstatement. to the insurer by any provision of this chapter, such right
When US life sought to rescind the contract on the ground of must be exercised previous to the commencement of an
concealment/misrepresentation, two years had not yet elapsed. action on the contract.
Hence, the contract can still be rescinded.
The defendant contended in the lower court and now contends in
G.R. No. L-27541 November 21, 1927 this court, that section 47 does not apply to the new matters
alleged in the special defense. If in legal effect defendant's special
defense is in the nature of an act to rescind "a contract of
TAN CHAY HENG vs. THE WEST COAST LIFE INSURANCE insurance," then such right must be exercised prior to an action
COMPANY enforce the contract.

Facts: Plaintiff alleges that he is of age and a resident of Bacolod, Issue: WON sec. 47 should apply?
Occidental Negros; that the defendant is a foreign insurance
corporation duly organized by the laws of the Philippines to engage
in the insurance business; that in the month of April, 1925, on his Held: No. In the instant case, it will be noted that even in its prayer,
application the defendant accepted and approved a life insurance the defendant does not seek to have the alleged insurance contract
policy for the sum of P10,000 in which the plaintiff was the sole rescinded. It denies that it ever made any contract of insurance on
beneficiary; that the policy was issued upon the payment by the the life of Tan Ceang or that any such a contract ever existed, and
said Tan Ceang of the first year's premium amounting to P936; that that is the question which it seeks to have litigated by its special
in and by its terms, the defendant agreed to pay the plaintiff as defense. In the very nature of things, if the defendant never made
beneficiary the amount of the policy upon the receipt of the proofs or entered into the contract in question, there is no contract to
of the death of the insured while the policy was in force; that rescind, and, hence, section 47 upon which the lower based its
without any premium being due or unpaid, Tan Ceang died on May decision in sustaining the demurrer does not apply. As stated, an
10, 1925; that in June, 1925, plaintiff submitted the proofs of the action to rescind a contract is founded upon and presupposes the
death of Tan Ceang with a claim for the payment of the policy existence of the contract which is sought to be rescinded. If all of
which the defendant refused to pay, for which he prays for a the material matters set forth and alleged in the defendant's
corresponding judgment, with legal interest from the date of the special plea are true, there was no valid contract of insurance, for
policy, and costs. the simple reason that the minds of the parties never met and
never agreed upon the terms and conditions of the contract. We are
clearly of the opinion that, if such matters are known to exist by a
In its defense, the defendant interposes that the insurance policy preponderance of the evidence, they would constitute a valid
on the life of Tan Ceang was obtained by the plaintiff in defense to plaintiff's cause of action. Upon the question as to
confabulation with one Go Chulian, Francisco Sanchez and Dr. V. S. whether or not they or are not true, we do not at this time have or
Locsin, of La Carlota thru fraud and deceit perpetrated against this express any opinion, but we are clear that section 47 does not
defendant. The lower court rendered judgment for the plaintiff for apply to the allegations made in the answer, and that the trial court
P10,000, with legal interest from January 4, 1926, and costs, to erred in sustaining the demurrer.
the Insurance Commissioner. After hearing the evidence of both
G.R. No. 48049 June 29, 1989 parties, the Insurance Commissioner rendered judgment dismissing
EMILIO TAN, JUANITO TAN, ALBERTO TAN and ARTURO TAN, petitioners' complaint. The Court of Appeals dismissed ' the
petitioners, petitioners' appeal from the Insurance Commissioner's decision for
vs. lack of merit.
THE COURT OF APPEALS and THE PHILIPPINE AMERICAN The CA concluded that respondent insurer has the right to
LIFE INSURANCE COMPANY, respondents. rescind the policy contract when insured is already dead. It also
GUTIERREZ, JR., J.: ruled that respondent insurer may be allowed to avoid the policy on
Facts: grounds of concealment by the deceased assured, is contrary to
Petitioners appeal from the Decision of the Insurance the provisions of the policy contract itself, as well as, of applicable
Commissioner dismissing herein petitioners' complaint against legal provisions and established jurisprudence.
respondent Philippine American Life Insurance Company for the The petitioners contend that the respondent company no
recovery of the proceeds of Policy No. 1082467 in the amount of P longer had the right to rescind the contract of insurance as
80,000.00. rescission must allegedly be done during the lifetime of the insured
Tan Lee Siong, father of herein petitioners, applied for life within two years and prior to the commencement of action.
insurance in the amount of P 80,000.00 with respondent company.
Said application was approved and Policy No. 1082467 was issued Issue: WON respondent company can rescind the contract of
effective November 6,1973, with petitioners the beneficiaries insurance on the ground of concealment
thereof. On April 26,1975, Tan Lee Siong died of hepatoma.
Petitioners then filed with respondent company their claim for the Held: Yes. Section 48 of the insurance code provides:
proceeds of the life insurance policy. However, respondent Whenever a right to rescind a contract of insurance is given
company denied petitioners' claim and rescinded the policy by to the insurer by any provision of this chapter, such
reason of the alleged misrepresentation and concealment of right must be exercised previous to the commencement
material facts made by the deceased Tan Lee Siong in his of an action on the contract.
application for insurance. The premiums paid on the policy were After a policy of life insurance made payable on the death of
thereupon refunded. the insured shall have been in force during the lifetime of
The evidence for respondent company shows that on the insured for a period of two years from the date of its
September 19,1972, the deceased was examined by Dr. Victoriano issue or of its last reinstatement, the insurer cannot prove
Lim and was found to be diabetic and hypertensive; that by that the policy is void ab initio or is rescindable by
January, 1973, the deceased was complaining of progressive weight reason of the fraudulent concealment or misrepresentation of
loss and abdominal pain and was diagnosed to be suffering from the insured or his agent.
hepatoma. Another physician, Dr. Wenceslao Vitug, testified that The so-called "incontestability clause" precludes the insurer
the deceased came to see him on December 14, 1973 for from raising the defenses of false representations or concealment
consolation and claimed to have been diabetic for five years. of material facts insofar as health and previous diseases are
Because of the concealment made by the deceased of his concerned if the insurance has been in force for at least two years
consultations and treatments for hypertension, diabetes and liver during the insured's lifetime. The phrase "during the lifetime" found
disorders, respondent company was thus misled into accepting the in Section 48 simply means that the policy is no longer considered
risk and approving his application as medically standard (and in force after the insured has died. The key phrase in the second
dispensing with further medical investigation and examination).For paragraph of Section 48 is "for a period of two years."
as long as no adverse medical history is revealed in the application The policy was issued on November 6,1973 and the insured
form, an applicant for insurance is presumed to be healthy and died on April 26,1975. The policy was thus in force for a period of
physically fit and no further medical investigation or examination is only one year and five months. Considering that the insured died
conducted by respondent company. before the two-year period had lapsed, respondent company is not,
Alleging that respondent company's refusal to pay them the therefore, barred from proving that the policy is void ab initio by
proceeds of the policy was unjustified and unreasonable, reason of the insured's fraudulent concealment or
petitioners filed a complaint against the former with the Office of misrepresentation. Moreover, respondent company rescinded the
contract of insurance and refunded the premiums paid on findings: 1. Sotero did not personally apply for insurance coverage,
September 11, 1975, previous to the commencement of this action as she was illiterate; 2. Sotero was sickly since 1990; 3. Sotero did
on November 27,1975. not have the financial capability to pay the insurance premiums on
After a policy of life insurance made payable on the death of Insurance Policy No. 747411; 4. Sotero did not sign the July 3, 1993
the insured shall have been in force during the lifetime of the application for insurance;9 and 5. Respondent was the one who
insured for a period of two years from the date of its issue or of its filed the insurance application, and x x x designated herself as the
last reinstatement, the insurer cannot prove that the policy is void beneficiary.
ab initio or is rescindable by reason of the fraudulent concealment For the above reasons, petitioner denied respondents claim on
or misrepresentation of the insured or his agent. April 16, 1997 and refunded the premiums paid on the policy. On
The petitioners argue that no evidence was presented to April 24, 1997, petitioner filed a civil case for rescission and/or
show that the medical terms were explained in a layman's annulment of the policy. The main thesis of the Complaint was that
language to the insured. They state that the insurer should have the policy was obtained by fraud, concealment and/or
presented its two medical field examiners as witnesses. Moreover, misrepresentation under the Insurance Code, which thus renders it
the petitioners allege that the policy intends that the medical voidable.
examination must be conducted before its issuance otherwise the Respondent filed a Motion to Dismiss claiming that petitioners
insurer "waives whatever imperfection by ratification." cause of action was barred by prescription pursuant to Section 48
There is no strong showing that the Court should apply the of the Insurance Code. The CA sustained the trial court. Applying
"fine print" or "contract of adhesion" rule in this case. There is no Section 48 to petitioners case, the CA held that petitioner may no
showing that the questions in the application form for insurance longer prove that the subject policy was void ab initio or rescindible
regarding the insured's medical history are in smaller print than the by reason of fraudulent concealment or misrepresentation after the
rest of the printed form or that they are designed in such a way as lapse of more than two years from its issuance. It ratiocinated that
to conceal from the applicant their importance. If a warning in bold petitioner was equipped with ample means to determine, within the
red letters or a boxed warning similar to that required for cigarette first two years of the policy, whether fraud, concealment or
advertisements by the Surgeon General of the United States is misrepresentation was present when the insurance coverage was
necessary, that is for Congress or the Insurance Commission to obtained. If it failed to do so within the statutory two-year period,
provide as protection against high pressure insurance then the insured must be protected and allowed to claim upon the
salesmanship. We are limited in this petition to ascertaining policy.
whether or not the respondent Court of Appeals committed ISSUE: Whether or not CA erred in applying the
reversible error. It is the petitioners' burden to show that the factual incontestability clause.
findings of the respondent court are not based on substantial RULING: NO. Section 48 serves a noble purpose, as it
evidence or that its conclusions are contrary to applicable law and regulates the actions of both the insurer and the insured. Under the
jurisprudence. They have failed to discharge that burden. provision, an insurer is given two years from the effectivity of a
WHEREFORE, the petition is hereby DENIED for lack of merit. The life insurance contract and while the insured is alive to discover or
questioned decision of the Court of Appeals is AFFIRMED. prove that the policy is void ab initio or is rescindible by reason of
the fraudulent concealment or misrepresentation of the insured or
MANILA BANKERS LIFE INSURANCE CORPORATION, his agent. After the two-year period lapses, or when the insured
Petitioner. vs. CRESENCIA P. ABAN, Respondent dies within the period, the insurer must make good on the policy,
G.R. No. 175666 July 29, 2013 even though the policy was obtained by fraud, concealment, or
DEL CASTILLO, J. misrepresentation. This is not to say that insurance fraud must be
FACTS: Delia Sotero took out a life insurance policy from rewarded, but that insurers who recklessly and indiscriminately
Manila Bankers Life Insurance Corporation, designating respondent solicit and obtain business must be penalized, for such recklessness
Cresencia P. Aban, her niece, as her beneficiary. On April 10, 1996, and lack of discrimination ultimately work to the detriment of bona
when the insurance policy had been in force for more than two fide takers of insurance and the public in general.
years and seven months, Sotero died. Respondent filed a claim for Section 48 regulates both the actions of the insurers and
the insurance proceeds on July 9, 1996. Petitioner conducted an prospective takers of life insurance. It gives insurers enough time to
investigation into the claim, and came out with the following inquire whether the policy was obtained by fraud, concealment, or
misrepresentation; on the other hand, it forewarns scheming After two years, the defenses of concealment or misrepresentation,
individuals that their attempts at insurance fraud would be timely no matter how patent or well-founded, will no longer lie.
uncovered thus deterring them from venturing into such nefarious As borne by the records, the policy was issued on August 30, 1993,
enterprise. At the same time, legitimate policy holders are the insured died on April 10, 1996, and the claim was denied on
absolutely protected from unwarranted denial of their claims or April 16, 1997. The insurance policy was thus in force for a period
delay in the collection of insurance proceeds occasioned by of 3 years, 7 months, and 24 days. Considering that the insured
allegations of fraud, concealment, or misrepresentation by insurers, died after the two-year period, the plaintiff-appellant is, therefore,
claims which may no longer be set up after the two-year period barred from proving that the policy is void ab initio by reason of the
expires as ordained under the law. insureds fraudulent concealment or misrepresentation or want of
Thus, the self-regulating feature of Section 48 lies in the fact that insurable interest on the part of the beneficiary, herein defendant-
both the insurer and the insured are given the assurance that any appellee.
dishonest scheme to obtain life insurance would be exposed, and insurers cannot be allowed to collect premiums on insurance
attempts at unduly denying a claim would be struck down. Life policies, use these amounts collected and invest the same through
insurance policies that pass the statutory two-year period are the years, generating profits and returns therefrom for their own
essentially treated as legitimate and beyond question, and the benefit, and thereafter conveniently deny insurance claims by
individuals who wield them are made secure by the thought that questioning the authority or integrity of their own agents or the
they will be paid promptly upon claim. In this manner, Section 48 insurance policies they issued to their premium-paying clients. This
contributes to the stability of the insurance industry. is exactly one of the schemes which Section 48 aims to prevent.
Section 48 prevents a situation where the insurer knowingly XXX The business of insurance is a highly regulated commercial
continues to accept annual premium payments on life insurance, activity in the country, and is imbued with public interest. "An
only to later on deny a claim on the policy on specious claims of insurance contract is a contract of adhesion which must be
fraudulent concealment and misrepresentation, such as what construed liberally in favor of the insured and strictly against the
obtains in the instant case. Thus, instead of conducting at the first insurer in order to safeguard the formers interest."
instance an investigation into the circumstances surrounding the
issuance of Insurance Policy No. 747411 which would have timely THE POLICY
exposed the supposed flaws and irregularities attending it as it now
professes, petitioner appears to have turned a blind eye and opted G.R. No. L-15895 November 29, 1920
instead to continue collecting the premiums on the policy. For RAFAEL ENRIQUEZ, as administrator of the estate of the late
nearly three years, petitioner collected the premiums and devoted Joaquin Ma. Herrer, plaintiff-appellant, vs.
the same to its own profit. It cannot now deny the claim when it is SUN LIFE ASSURANCE COMPANY OF CANADA, defendant-
called to account. Section 48 must be applied to it with full force appellee.
and effect.
The insurer is deemed to have the necessary facilities to discover This is an action brought by the plaintiff ad administrator of the
such fraudulent concealment or misrepresentation within a period estate of the late Joaquin Ma. Herrer to recover from the defendant
of two (2) years. It is not fair for the insurer to collect the premiums life insurance company the sum of pesos 6,000 paid by the
as long as the insured is still alive, only to raise the issue of deceased for a life annuity. The trial court gave judgment for the
fraudulent concealment or misrepresentation when the insured dies defendant. Plaintiff appeals.
in order to defeat the right of the beneficiary to recover under the FACTS: On September 24, 1917, Joaquin Herrer made application to
policy. the Sun Life Assurance Company of Canada through its office in
At least two (2) years from the issuance of the policy or its last Manila for a life annuity. Two days later he paid the sum of P6,000
reinstatement, the beneficiary is given the stability to recover to the manager of the company's Manila office and was given a
under the policy when the insured dies. The provision also makes provisional receipt.
clear when the two-year period should commence in case the The application was immediately forwarded to the head office of
policy should lapse and is reinstated, that is, from the date of the the company at Montreal, Canada. On November 26, 1917, the
last reinstatement. head office gave notice of acceptance by cable to Manila. (Whether
on the same day the cable was received notice was sent by the
Manila office of Herrer that the application had been accepted, is a to the provisional receipt, three things had to be accomplished by
disputed point, which will be discussed later.) On December 4, the insurance company before there was a contract: (1) There had
1917, the policy was issued at Montreal. On December 18, 1917, to be a medical examination of the applicant; (2) there had to be
attorney Aurelio A. Torres wrote to the Manila office of the company approval of the application by the head office of the company; and
stating that Herrer desired to withdraw his application. The (3) this approval had in some way to be communicated by the
following day the local office replied to Mr. Torres, stating that the company to the applicant. The further admitted facts are that the
policy had been issued, and called attention to the notification of head office in Montreal did accept the application, did cable the
November 26, 1917. This letter was received by Mr. Torres on the Manila office to that effect, did actually issue the policy and did,
morning of December 21, 1917. Mr. Herrer died on December 20, through its agent in Manila, actually write the letter of notification
1917. and place it in the usual channels for transmission to the
addressee. The fact as to the letter of notification thus fails to
ISSUE: Whether or not Herrer received notice of acceptance of his concur with the essential elements of the general rule pertaining to
application. the mailing and delivery of mail matter as announced by the
HELD: No. American courts, namely, when a letter or other mail matter is
The chief clerk of the Manila office of the Sun Life Assurance addressed and mailed with postage prepaid there is a rebuttable
Company of Canada at the time of the trial testified that he presumption of fact that it was received by the addressee as soon
prepared the letter dated November 26, 1917, and handed it to the as it could have been transmitted to him in the ordinary course of
local manager for signature. The witness admitted on cross- the mails. But if any one of these elemental facts fails to appear, it
examination that after preparing the letter and giving it to the is fatal to the presumption. For instance, a letter will not be
manager, he new nothing of what became of it. The local manager, presumed to have been received by the addressee unless it is
Mr. White, testified to having received the cablegram accepting the shown that it was deposited in the post-office, properly addressed
application of Mr. Herrer from the home office on November 26, and stamped. (See 22 C.J., 96, and 49 L. R. A. [N. S.], pp. 458, et
1917. He said that on the same day he signed a letter notifying Mr. seq., notes.)
Herrer of this acceptance. The witness further said that letters, We hold that the contract for a life annuity in the case at bar was
after being signed, were sent to the chief clerk and placed on the not perfected because it has not been proved satisfactorily that the
mailing desk for transmission. The witness could not tell if the letter acceptance of the application ever came to the knowledge of the
had actually been placed in the mails. Mr. Tuason, who was the applicant.
chief clerk, on November 26, 1917, was not called as a witness. For
the defense, attorney Manuel Torres testified to having prepared
the will of Joaquin Ma. Herrer, that on this occasion, Mr. Herrer VIRGINIA A. PEREZ, petitioner, vs. COURT OF APPEALS and
mentioned his application for a life annuity, and that he said that BF LIFEMAN INSURANCE CORPORATION, respondents.
the only document relating to the transaction in his possession was
the provisional receipt. Rafael Enriquez, the administrator of the G.R. No. 112329. January 28, 2000
estate, testified that he had gone through the effects of the
deceased and had found no letter of notification from the insurance
company to Mr. Herrer. A contract of insurance, like all other contracts, must be assented
Our deduction from the evidence on this issue must be that the to by both parties, either in person or through their agents and so
letter of November 26, 1917, notifying Mr. Herrer that his long as an application for insurance has not been either accepted
application had been accepted, was prepared and signed in the or rejected, it is merely a proposal or an offer to make a contract.
local office of the insurance company, was placed in the ordinary
channels for transmission, but as far as we know, was never Facts:
actually mailed and thus was never received by the applicant.
The law applicable to the case is found to be the second paragraph Primitivo B. Perez had been insured with the BF Lifeman Insurance
of article 1262 of the Civil Code providing that an acceptance made Corporation since 1980 for P20,000.00. Sometime in October 1987,
by letter shall not bind the person making the offer except from the an agent of the insurance corporation, Rodolfo Lalog, visited Perez
time it came to his knowledge. The pertinent fact is, that according in Guinayangan, Quezon and convinced him to apply for additional
insurance coverage of P50,000.00, to avail of the ongoing The RTC rendered a decision in favor of Virginia. The premium for
promotional discount of P400.00 if the premium were paid annually. the additional insurance of P50,000.00 had been fully paid and
even if the sum of P2,075.00 were to be considered merely as
On October 20, 1987, Primitivo B. Perez accomplished an partial payment, the same does not affect the validity of the policy.
application form for the additional insurance coverage of The trial court further stated that the deceased had fully complied
P50,000.00. On the same day, petitioner Virginia A. Perez, with the requirements of the insurance company. He paid, signed
Primitivos wife, paid P2,075.00 to Lalog. The receipt issued by the application form and passed the medical examination. He
Lalog indicated the amount received was a "deposit." should not be made to suffer the subsequent delay in the
Unfortunately, Lalog lost the application form accomplished by transmittal of his application form to private respondents head
Perez and so he asked the latter to fill up another application form. office since these were no longer within his control.
On November 1, 1987, Perez was made to undergo the required
medical examination, which he passed. The CA reversed the decision of the trial court saying that the
insurance contract for P50,000.00 could not have been perfected
Pursuant to the established procedure of the company, Lalog since at the time that the policy was issued, Primitivo was already
forwarded the application for additional insurance of Perez, dead. The Court of Appeals held that the contract of insurance had
together with all its supporting papers, to the office of BF Lifeman to be assented to by both parties and so long as the application for
Insurance Corporation at Gumaca, Quezon which office was insurance has not been either accepted or rejected, it is merely an
supposed to forward the papers to the Manila office. offer or proposal to make a contract.

On November 25, 1987, Perez died in an accident. He was riding in Issue:


a banca which capsized during a storm. At the time of his death, his
application papers for the additional insurance of P50,000.00 were 1. Whether, as between Primitivo and BF Lifeman Insurance Corp,
still with the Gumaca office. Lalog testified that when he went to there was a perfected contract of insurance.
follow up the papers, he found them still in the Gumaca office and
so he personally brought the papers to the Manila office of BF 2. Whether the condition that the policy issued by the corporation
Lifeman Insurance Corporation. It was only on November 27, 1987 be delivered and received by the applicant in good health, is
that said papers were received in Manila. Without knowing that potestative, being dependent upon the will of the insurance
Perez died on November 25, 1987, BF Lifeman Insurance company, and is therefore null and void.
Corporation approved the application and issued the corresponding
policy for the P50,000.00 on December 2, 1987. Ruling:

Petitioner Virginia Perez went to Manila to claim the benefits under 1. None. When Primitivo filed an application for insurance, paid
the insurance policies of the deceased. She was paid P40,000.00 P2,075.00 and submitted the results of his medical examination, his
under the first insurance policy, but was not paid under the application was subject to the acceptance of private respondent BF
additional policy coverage. In the letter of BF Lifeman to Virginia, it Lifeman Insurance Corporation. The perfection of the contract of
maintained that the insurance for P50,000.00 had not been insurance between the deceased and respondent corporation was
perfected at the time of the death of Primitivo Perez. Consequently, further conditioned upon compliance with the following requisites
the insurance company refunded the amount of P2,075.00 which stated in the application form:
Virginia Perez had paid.
"there shall be no contract of insurance unless and until a policy is
Petitioner Virginia A. Perez, on the other hand, averred that the issued on this application and that the said policy shall not take
deceased had fulfilled all his prestations under the contract and all effect until the premium has been paid and the policy delivered to
the elements of a valid contract are present. and accepted by me/us in person while I/We, am/are in good
health."
The assent of private respondent BF Lifeman Insurance Corporation be done, nothing to be completed, nothing to be passed upon, or
therefore was not given when it merely received the application determined, before it shall take effect. There can be no contract of
form and all the requisite supporting papers of the applicant. Its insurance unless the minds of the parties have met.
assent was given when it issues a corresponding policy to the
applicant. Also, it when Primitivo died on November 25, 1987, his
application papers for additional insurance coverage were still with [G.R. No. 119176. March 19, 2002]
the branch office of respondent corporation in Gumaca and it was COMMISSIONER OF INTERNAL REVENUE, petitioner, vs.
only two days later when Lalog personally delivered the application LINCOLN PHILIPPINE LIFE INSURANCE COMPANY, INC. (now
papers to the head office in Manila. Consequently, there was JARDINE-CMA LIFE INSURANCE COMPANY, INC.) and THE
absolutely no way the acceptance of the application could have COURT OF APPEALS, respondents.
been communicated to the applicant for the latter to accept
inasmuch as the applicant at the time was already dead. Facts: Prior to 1984, Lincoln Philippine Life Insurance Company,
Inc. (now called Jardine-CMA Life Insurance Company, Inc.) used to
2. The SC did not agree. In the case at bar, the following conditions issue policies called Junior Estate Builder Policy. A clause therein
were imposed by the respondent company for the perfection of the provides for an automatic increase in the amount of life insurance
contract of insurance: coverage upon attainment of a certain age by the insured without
the need of issuing a new policy. The clause was to take effect in
1. a policy must have been issued; the year 1984. Documentary stamp taxes due on the policy were
paid by Lincoln Philippine only on the initial sum assured.
2. the premiums paid; and When the clause became effective in 1984, the Commissioner of
Internal Revenue assessed an additional tax on the increased
amount of the coverage of the said policies. Said tax was to cover
3. the policy must have been delivered to and accepted by the deficiency documentary stamps tax for said year. The Court of
the applicant while he is in good health. Appeals ruled that there is only one policy and the automatic
increase is not a separate policy; that said increase of coverage is
The condition imposed by the corporation that the policy must have not covered by another documentary stamp tax.
been delivered to and accepted by the applicant while he is in good ISSUE: Whether or not there is only one policy.
health can hardly be considered as a potestative or facultative HELD: Yes. Section 49, Title VI of the Insurance Code defines an
condition. On the contrary, the health of the applicant at the time insurance policy as the written instrument in which a contract of
of the delivery of the policy is beyond the control or will of the insurance is set forth. Section 50 of the same Code provides that
insurance company. Rather, the condition is a suspensive one the policy, which is required to be in printed form, may contain any
whereby the acquisition of rights depends upon the happening of word, phrase, clause, mark, sign, symbol, signature, number, or
an event which constitutes the condition. In this case, the word necessary to complete the contract of insurance. It is thus
suspensive condition was the policy must have been delivered and clear that any rider, clause, warranty or endorsement pasted or
accepted by the applicant while he is in good health. There was attached to the policy is considered part of such policy or contract
non-fulfillment of the condition, however, inasmuch as the of insurance.
applicant was already dead at the time the policy was issued. The subject insurance policy at the time it was issued contained an
Hence, the non-fulfillment of the condition resulted in the non- automatic increase clause. Although the clause was to take effect
perfection of the contract. only in 1984, it was written into the policy at the time of its
issuance. The distinctive feature of the junior estate builder
As stated above, a contract of insurance, like other contracts, must policy called the automatic increase clause already formed part
be assented to by both parties either in person or by their agents. and parcel of the insurance contract, hence, there was no need for
So long as an application for insurance has not been either an execution of a separate agreement for the increase in the
accepted or rejected, it is merely an offer or proposal to make a coverage that took effect in 1984 when the assured reached a
contract. The contract, to be binding from the date of application, certain age.
must have been a completed contract, one that leaves nothing to
The said increase however is imposable with documentary stamp the company shall confirm this agreement by issuing
taxes. The original documentary stamps tax paid by Lincoln a policy on said application xxx.
Philippine covers the original amount of the policies without the Should the company NOT issue such a policy, then this
projected increase. The said increase was already definite at the agreement shall be null and void ab initio and the Company
time of the issuance of the policy. Thus, the amount insured by the shall be held not to have been on the risk at all, but in such
policy at the time of its issuance necessarily included the additional case, the amount herein shall be returned
sum covered by the automatic increase clause because it was Lim died on Aug. 23, 1917 after the issuance of the
already determinable at the time the transaction was entered into provisional policy but before the approval of the application
and formed part of the policy. by the home office of the insurance company.
While tax avoidance schemes and arrangements are not prohibited, The instant action is brought by the beneficiary, his wife,
tax laws cannot be circumvented in order to evade the payment of Pilar, to recover from Sun Life the sum of 5k
just taxes. In the case at bar, to claim that the increase in the
amount insured (by virtue of the automatic increase clause ISSUE: WON such provisional policy be considered as the insurance
incorporated into the policy at the time of issuance) should not be policy itself
included in the computation of the documentary stamp taxes due
on the policy would be a clear evasion of the law requiring that the HELD:
tax be computed on the basis of the amount insured by the policy. NO
The contract of insurance was not consummated by the
LIM vs. SUN LIFE of CANADA parties. The above quoted agreement clearly stated that
the agreement should NOT go into effect until the home
office of the Company shall confirm it by issuing a policy.
Doctrine:
It was nothing but an acknowledgment by the Company
Binding-Slip doesnt insure itself
that it has received a sum of money agreed upon as the
When an agreement is made between the applicant and the
first years premium upon a policy to be issued upon the
agent whether by signing an application containing such application if it is accepted by the Company.
condition or otherwise, that no liability shall attach until the
When an agreement is made between the applicant and the
principal approves the risk and a receipt is given by the
agent whether by signing an application containing such
agent, such acceptance is merely conditional and is
condition or otherwise, that no liability shall attach until the
subordinated to the companys act in approving or rejecting;
principal approves the risk and a receipt is given by the
so in life insurance a binding slip or receipt does not
agent, such acceptance is merely conditional and is
insure itself.
subordinated to the companys act in approving or
rejecting; so in life insurance a binding slip or receipt does
FACTS:
not insure itself.
On July 6, 1917, Luis Lim Y Garcia of Zamboanga applied for
a policy of life insurance with Sunlife in the amount of 5k
He designated his wife Pilar Lim as the beneficiary. The first G.R. No. L-38613 February 25, 1982
premium of P433 was paid by Lim and company issued a PACIFIC TIMBER EXPORT CORPORATION, petitioner,
provisional policy vs.
Such policy contained the following provisions THE HONORABLE COURT OF APPEALS and WORKMEN'S
o xx the abovementioned life is to be assured in INSURANCE COMPANY, INC., respondents.
accordance with the terms and conditions contained DE CASTRO, ** J.:
or inserted by the Company in the policy which may
be granted by it in this particular case for 4 months
only from the date of the application, PROVIDED that Facts
Pacific timber secured temporary insurance from the Workemens policies, thereby leaving no account unpaid by petitioner due on
Insurance Co. for its exportation of logs to Japan. Workmen issued the insurance coverage, which must be deemed to include the
on said date Cover Note 1010 insuring said cargo. The regular Cover Note. If the Note is to be treated as a separate policy instead
marine cargo 2 policies were issued by the defendant in favor of of integrating it to the regular policies subsequently issued, the
the plaintiff. The total cargo insured under the two marine policies purpose and function of the Cover Note would be set at naught or
accordingly consisted of 1,395 logs. rendered meaningless, for it is in a real sense a contract, not a
mere application for insurance which is a mere offer.
After the issuance of a Cover Note but before the issuance of the
two marine policies, some of the logs intended to be exported were The adjuster went as far as submitting his report to respondent, as
lost during loading operations in the Diapitan Bay due to a bad well as its computation of respondent's liability on the insurance
weather. coverage. This coverage could not have been no other than what
was stipulated in the Cover Note, for no loss or damage had to be
The plaintiff informed the defendant about the loss of several logs. assessed on the coverage arising from the marine insurance
The defendant wrote the plaintiff denying the latter's claim that the policies. For obvious reasons, it was not necessary to ask petitioner
loss of 30 pieces of logs is not covered by 2 Marine Policies. to pay premium on the Cover Note, for the loss insured against
However, the defendants did not deny that the said loss may be having already occurred, the more practical procedure is simply to
covered under the Cover Note. Unfortunately, the cover note is null deduct the premium from the amount due the petitioner on the
and void for lack of valuable consideration. Cover Note. The non-payment of premium on the Cover Note is,
therefore, no cause for the petitioner to lose what is due it as if
Issue there had been payment of premium, for non-payment by it was
Is the cover note null and void for lack of valuable consideration? not chargeable against its fault. Had all the logs been lost during
the loading operations, but after the issuance of the Cover Note,
Held: liability on the note would have already arisen even before
payment of premium. This is how the cover note as a "binder"
No. Petitioner contends that the Cover Note was issued with a should legally operate otherwise, it would serve no practical
consideration when, by express stipulation, the cover note is made purpose in the realm of commerce, and is supported by the
subject to the terms and conditions of the marine policies, and the doctrine that where a policy is delivered without requiring payment
payment of premiums is one of the terms of the policies. From this of the premium, the presumption is that a credit was intended and
undisputed fact, We uphold petitioner's submission that the Cover policy is valid.
Note was not without consideration for which the respondent court
held the Cover Note as null and void, and denied recovery G.R. No. L-109937 March 21, 1994
therefrom. The fact that no separate premium was paid on the
Cover Note before the loss insured against occurred, does not
militate against the validity of petitioner's contention, for no such DEVELOPMENT BANK OF THE PHILIPPINES, petitioner,
premium could have been paid, since by the nature of the Cover vs.
Note, it did not contain, as all Cover Notes do not contain COURT OF APPEALS and the ESTATE OF THE LATE JUAN B.
particulars of the shipment that would serve as basis for the
computation of the premiums. As a logical consequence, no DANS, represented by CANDIDA G. DANS, and the DBP
separate premiums are intended or required to be paid on a Cover MORTGAGE REDEMPTION INSURANCE POOL, respondents.
Note. This is a fact admitted by an official of respondent company,
Juan Jose Camacho, in charge of issuing cover notes of the
QUIASON, J.:
respondent company (p. 33, tsn, September 24, 1965).

At any rate, it is not disputed that petitioner paid in full all the 1. Juan B. Dans, together with his wife Candida, his son and
premiums as called for by the statement issued by private daughter-in-law, applied for a loan of P500,000.00 DBP, Basilan
respondent after the issuance of the two regular marine insurance
Branch. As the principal mortgagor, Dans, then 76 years of age, premium thereon. Respondent Estate therefore prayed: (1) that the
was advised by DBP to obtain a mortgage redemption insurance sum of P139,500.00, which it paid under protest for the loan, be
(MRI) with the DBP Mortgage Redemption Insurance Pool (DBP MRI reimbursed; (2) that the mortgage debt of the deceased be
Pool). declared fully paid; and (3) that damages be awarded.

2. A loan, in the reduced amount of P300,000.00, was approved by ISSUE:


DBP on August 4, 1987 and released on August 11, 1987. From the 1. Whether or not there was a perfected contract of insurance
proceeds of the loan, DBP deducted the amount of P1,476.00 as between DBP MRI Pool and Juan Dans
payment for the MRI premium. On August 15, 1987, Dans 2. Whether or not DBP is liable
accomplished and submitted the "MRI Application for Insurance"
and the "Health Statement for DBP MRI Pool." HELD:
1. NO. When Dans applied for MRI, he filled up and personally
3. On August 20, 1987, the MRI premium of Dans, less the DBP signed a "Health Statement for DBP MRI Pool" (Exh. "5-Bank") with
service fee of 10 percent, was credited by DBP to the savings the following declaration:
account of the DBP MRI Pool. Accordingly, the DBP MRI Pool was
advised of the credit. I hereby declare and agree that all the statements and answers
On September 3, 1987, Dans died of cardiac arrest. The DBP, upon contained herein are true, complete and correct to the best of my
notice, relayed this information to the DBP MRI Pool. On September knowledge and belief and form part of my application for insurance.
23, 1987, the DBP MRI Pool notified DBP that Dans was not eligible It is understood and agreed that no insurance coverage shall be
for MRI coverage, being over the acceptance age limit of 60 years effected unless and until this application is approved and the full
at the time of application. premium is paid during my continued good health (Records, p. 40).

4. DBP apprised Candida Dans of the disapproval of her late Under the aforementioned provisions, the MRI coverage shall take
husband's MRI application. The DBP offered to refund the premium effect: (1) when the application shall be approved by the insurance
of P1,476.00 which the deceased had paid, but Candida Dans pool; and (2) when the full premium is paid during the continued
refused to accept the same, demanding payment of the face value good health of the applicant. These two conditions, being joined
of the MRI or an amount equivalent to the loan. She, likewise, conjunctively, must concur.
refused to accept an ex gratia settlement of P30,000.00, which the
DBP later offered. Undisputably, the power to approve MRI applications is lodged with
the DBP MRI Pool. The pool, however, did not approve the
5. Respondent Estate, through Candida Dans as administratrix, filed application of Dans. There is also no showing that it accepted the
a complaint with the RTC Basilan, against DBP and the insurance sum of P1,476.00, which DBP credited to its account with full
pool for "Collection of Sum of Money with Damages." Respondent knowledge that it was payment for Dan's premium. There was, as a
Estate alleged that Dans became insured by the DBP MRI Pool when result, no perfected contract of insurance; hence, the DBP MRI Pool
DBP, with full knowledge of Dans' age at the time of application, cannot be held liable on a contract that does not exist.
required him to apply for MRI, and later collected the insurance
2. YES. It was DBP, as a matter of policy and practice, that required The liability of an agent who exceeds the scope of his authority
Dans, the borrower, to secure MRI coverage. Instead of allowing depends upon whether the third person is aware of the limits of the
Dans to look for his own insurance carrier or some other form of agent's powers. There is no showing that Dans knew of the
insurance policy, DBP compelled him to apply with the DBP MRI limitation on DBP's authority to solicit applications for MRI.
Pool for MRI coverage. When Dan's loan was released on August 11,
1987, DBP already deducted from the proceeds thereof the MRI The DBP's liability, however, cannot be for the entire value of the
premium. Four days latter, DBP made Dans fill up and sign his insurance policy. To assume that were it not for DBP's concealment
application for MRI, as well as his health statement. The DBP later of the limits of its authority, Dans would have secured an MRI from
submitted both the application form and health statement to the another insurance company, and therefore would have been fully
DBP MRI Pool at the DBP Main Building, Makati Metro Manila. As insured by the time he died, is highly speculative. Considering his
service fee, DBP deducted 10 percent of the premium collected by advanced age, there is no absolute certainty that Dans could obtain
it from Dans. an insurance coverage from another company. It must also be
noted that Dans died almost immediately, i.e., on the nineteenth
In dealing with Dans, DBP was wearing two legal hats: the first as a day after applying for the MRI, and on the twenty-third day from the
lender, and the second as an insurance agent. date of release of his loan.

As an insurance agent, DBP made Dans go through the motion of While Dans is not entitled to compensatory damages, he is entitled
applying for said insurance, thereby leading him and his family to to moral damages. No proof of pecuniary loss is required in the
believe that they had already fulfilled all the requirements for the assessment of said kind of damages (Civil Code of Philippines, Art.
MRI and that the issuance of their policy was forthcoming. 2216). The same may be recovered in acts referred to in Article
Apparently, DBP had full knowledge that Dan's application was 2219 of the Civil Code.
never going to be approved. The maximum age for MRI acceptance
is 60 years as clearly and specifically provided in Article 1 of the G.R. No. L-20853 May 29, 1967
Group Mortgage Redemption Insurance Policy signed in 1984 by all
BONIFACIO BROS., INC., ET AL. vs. ENRIQUE MORA, ET AL.
the insurance companies concerned (Exh. "1-Pool").
Under Article 1987 of the Civil Code of the Philippines, "the agent Facts: Enrique Mora, owner of Oldsmobile sedan model 1956
who acts as such is not personally liable to the party with whom he mortgaged the same to the H.S. Reyes, Inc., with the condition that
contracts, unless he expressly binds himself or exceeds the limits of the former would insure the automobile with the latter as
beneficiary. The automobile was thereafter insured on June 23,
his authority without giving such party sufficient notice of his 1959 with the State Bonding & Insurance Co., Inc., and motor car
powers." insurance policy was issued to Enrique Mora.
The DBP is not authorized to accept applications for MRI when its
clients are more than 60 years of age (Exh. "1-Pool"). Knowing all During the effectivity of the insurance contract, the car met with an
accident. The insurance company then assigned the accident to the
the while that Dans was ineligible for MRI coverage because of his Bayne Adjustment Co. for investigation and appraisal of the
advanced age, DBP exceeded the scope of its authority when it damage. Enrique Mora, without the knowledge and consent of the
accepted Dan's application for MRI by collecting the insurance H.S. Reyes, Inc., authorized the Bonifacio Bros. Inc. to furnish the
labor and materials, some of which were supplied by the Ayala Auto
premium, and deducting its agent's commission and service fee.
Parts Co. For the cost of labor and materials, Enrique Mora was or provision thereof from which we can infer that there is an
billed at P2,102.73 through the H.H. Bayne Adjustment Co. The obligation on the part of the insurance company to pay the cost of
insurance company drew a check in the amount of P2,002.73, as repairs directly to them. It is fundamental that contracts take effect
proceeds of the insurance policy, payable to the order of Enrique only between the parties thereto, except in some specific instances
Mora or H.S. Reyes,. Inc., and entrusted the check to the H.H. provided by law where the contract contains some stipulation in
Bayne Adjustment Co. for disposition and delivery to the proper favor of a third person. Such stipulation is known as stipulation pour
party. In the meantime, the car was delivered to Enrique Mora autrui or a provision in favor of a third person not a pay to the
without the consent of the H.S. Reyes, Inc., and without payment to contract. Under this doctrine, a third person is allowed to avail
the Bonifacio Bros. Inc. and the Ayala Auto Parts Co. of the cost of himself of a benefit granted to him by the terms of the contract,
repairs and materials. provided that the contracting parties have clearly and deliberately
conferred a favor upon such person. Consequently, a third person
Upon the theory that the insurance proceeds should be paid not a party to the contract has no action against the parties
directly to them, the Bonifacio Bros. Inc. and the Ayala Auto Parts thereto, and cannot generally demand the enforcement of the
Co. filed on May 8, 1961 a complaint with the Municipal Court of same. The question of whether a third person has an enforcible
Manila against Enrique Mora and the State Bonding & Insurance interest in a contract, must be settled by determining whether the
Co., Inc. for the collection of the sum of P2,002.73 The insurance contracting parties intended to tender him such an interest by
company filed its answer with a counterclaim for interpleader, deliberately inserting terms in their agreement with the avowed
requiring the Bonifacio Bros. Inc. and the H.S. Reyes, Inc. to purpose of conferring a favor upon such third person. In this
interplead in order to determine who has better right to the connection, this Court has laid down the rule that the fairest test to
insurance proceeds in question. Enrique Mora was declared in determine whether the interest of a third person in a contract is a
default for failure to appear at the hearing, and evidence against stipulation pour autrui or merely an incidental interest, is to rely
him was received ex parte. However, the counsel for the Bonifacio upon the intention of the parties as disclosed by their contract. In
Bros. Inc., Ayala Auto Parts Co. and State Bonding & Insurance Co. the instant case the insurance contract does not contain any words
Inc. submitted a stipulation of facts, on the basis of which are or clauses to disclose an intent to give any benefit to any
Municipal Court rendered a decision declaring the H.S. Reyes, Inc. repairmen or materialmen in case of repair of the car in question.
as having a better right to the disputed amount and ordering State The parties to the insurance contract omitted such stipulation,
Bonding & Insurance Co. Inc. to pay to the H. S. Reyes, Inc. the said which is a circumstance that supports the said conclusion. On the
sum of P2,002.73. From this decision, the appellants elevated the other hand, the "loss payable" clause of the insurance policy
case to the Court of First Instance of Manila which affirmed the stipulates that "Loss, if any, is payable to H.S. Reyes, Inc."
decision of the MTC. Hence, this appeal. indicating that it was only the H.S. Reyes, Inc. which they intended
to benefit.
Issue: WON there is privity of contract between the Bonifacio Bros.
Inc. and the Ayala Auto Parts Co. on the one hand and the As regards paragraph 4 of the insurance contract, a perusal thereof
insurance company on the other? would show that instead of establishing privity between the
appellants and the insurance company, such stipulation merely
Held: No. The appellants seek to recover the insurance proceeds, establishes the procedure that the insured has to follow in order to
and for this purpose, they rely upon paragraph 4 of the insurance be entitled to indemnity for repair. This paragraph therefore should
contract document executed by and between the State Bonding & not be construed as bringing into existence in favor of the
Insurance Company, Inc. and Enrique Mora which provides that "the appellants a right of action against the insurance company as such
insured may authorize the repair of the Motor Vehicle necessitated intention can never be inferred therefrom.
by damage for which the company may be liable under the policy
provided that (a) the estimated cost of such repair does not exceed Another cogent reason for not recognizing a right of action by the
the Authorized Repair Limit, and (b) a detailed estimate of the cost appellants against the insurance company is that "a policy of
is forwarded to the company without delay."The appellants are not insurance is a distinct and independent contract between the
mentioned in the contract as parties thereto nor is there any clause insured and insurer, and third persons have no right either in a
court of equity, or in a court of law, to the proceeds of it, unless 4) TRIAL COURT Ruled in favour of the Coquias. Hence, this appeal
there be some contract of trust, expressed or implied between the by the Fieldmens Insurance Co.
insured and third person." In this case, no contract of trust,
expressed or implied exists. We, therefore, agree with the trial ISSUES:
court that no cause of action exists in favor of the appellants in so 1) Whether or not the Coquias have no contractual relation with the
far as the proceeds of insurance are concerned. The appellants' Fieldmens Insurance Co.?
claim, if at all, is merely equitable in nature and must be made 2) Whether or not the Insured has not complied with the provisions
effective through Enrique Mora who entered into a contract with the of the policy concerning arbitration?
Bonifacio Bros. Inc. This conclusion is deducible not only from the
principle governing the operation and effect of insurance contracts HELD:
in general, but is clearly covered by the express provisions of
section 50 of the Insurance Act which read: 1) NO, the Coquias HAVE a contractual relation with the Fieldmens
Insurance Co by virtue of a contract pour autrui.
The insurance shall be applied exclusively to the proper GR: Only parties to a contract may bring an action based thereon
interests of the person in whose name it is made unless EXC: Art. 1311, CC (Contracts pour autrui): If a contract should
otherwise specified in the policy. contain some stipulation in favor of a third person, he may demand
its fulfillment provided he communicated his acceptance to the
MELECIO COQUIA, MARIA ESPANUEVA and MANILA YELLOW obligor before its revocation. A mere incidental benefit or interest of
TAXICAB CO., INC., vs. FIELDMEN'S INSURANCE CO., INC. a person is not sufficient. The contracting parties must have clearly
CONCEPCION, C.J.: and deliberately conferred a favor upon a third person.

FACTS: Dec. 1, 1961 Fieldmens Insurance Co., issued in favour of Pursuant to the stipulations in the policy, the Company "will
Manila Yellow Taxicab Co., Inc. a common carrier accident insurance indemnify any authorized Driver who is driving the Motor Vehicle"
policy, covering the period from December 1, 1961 to December 1, of the Insured and, in the event of death of said driver, the
1962. It was stipulated in said policy that: The Company will, Company shall, likewise, "indemnify his personal representatives."
subject to the Limits of Liability and under the Terms of this Policy, In fact, the Company "may, at its option, make indemnity payable
indemnify the Insured in the event of accident caused by or arising directly to the claimants or heirs of claimants ... it being the true
out of the use of Motor Vehicle against all sums which the Insured intention of this Policy to protect ... the liabilities of the Insured
will become legally liable to pay in respect of: Death or bodily injury towards the passengers of the Motor Vehicle and the Public" in
to any fare-paying passenger including the Driver ... who is riding in other words, third parties.
the Motor Vehicle insured at the time of accident or injury. Thus, the policy under consideration is typical of contracts
pour autrui, this character being made more manifest by the fact
2) Feb. 10, 1962 A taxicab of the Manila Yellow Taxicab Co., Inc. that the deceased driver paid fifty percent (50%) of the
(insured), driven by Carlito Coquia, met a vehicular accident at corresponding premiums, which were deducted from his weekly
Mangaldan, Pangasinan, in consequence of which Carlito died. The commissions. Under these conditions, it is clear that the Coquias
Insured filed therefor a claim for P5,000.00 to which the Company who, admittedly, are the sole heirs of the deceased have a direct
replied with an offer to pay P2,000.00, by way of compromise. The cause of action against the Company, and, since they could have
Insured rejected the same and made a counter-offer for P4,000.00, maintained this action by themselves, without the assistance of the
but the Company did not accept it. Insured, it goes without saying that they could and did properly join
the latter in filing the complaint herein.
3) Sept. 18, 1962 The Insured and Carlito's parents filed a
complaint against the Company to collect the proceeds of the 2) NO, both the acts or omissions of the Insured and the Company
aforementioned policy. In its answer, the Company admitted the constituted a waiver of their respective right to demand arbitration.
existence thereof, but pleaded lack of cause of action on the part of Section 17 of the Policy reads:
the plaintiffs. If any difference or dispute shall arise with respect to the
amount of the Company's liability under this Policy, the
same shall be referred to the decision of a single arbitrator 1. The Company, will, subject to the limits of liability,
to be agreed upon by both parties or failing such agreement indemnify the Insured in the event of accident caused by or
of a single arbitrator, to the decision of two arbitrators, one arising out of the use of the Motor Vehicle/s or in connection
to be appointed in writing by each of the parties within one with the loading or unloading of the Motor Vehicle/s, against
calendar month after having been required in writing so to all sums including claimant's costs and expenses which the
do by either of the parties and in case of disagreement Insured shall become legally liable to pay in respect of:
between the arbitrators, to the decision of an umpire who
shall have been appointed in writing by the arbitrators a. death of or bodily injury to any person
before entering on the reference and the costs of and
incident to the reference shall be dealt with in the Award. b. damage to property
And it is hereby expressly stipulated and declared that it
shall be a condition precedent to any right of action or suit
upon this Policy that the award by such arbitrator, E. Action Against Company
arbitrators or umpire of the amount of the Company's
liability hereunder if disputed shall be first obtained. No action shall lie against the Company unless, as a
condition precedent thereto, the Insured shall have fully
The record shows, however, that none of the parties to the contract complied with all of the terms of this Policy, nor until the
invoked Sec. 17 of the Policy, or made any reference to arbitration, amount of the Insured's obligation to pay shall have been
during the negotiations preceding the institution of the present finally determined either by judgment against the Insured
case. In fact, counsel for both parties stipulated, in the trial court, after actual trial or by written agreement of the Insured, the
that none of them had, at any time during said negotiations, even claimant, and the Company.
suggested the settlement of the issue between them by arbitration,
as provided in said section. Their aforementioned acts or omissions Any person or organization or the legal representative
had the effect of a waiver of their respective right to demand an thereof who has secured such judgment or written
arbitration. agreement shall thereafter be entitled to recover under this
policy to the extent of the insurance afforded by the Policy.
Nothing contained in this policy shall give any person or
DIONISIA, EULOGIO, MARINA, GUILLERMO and NORBERTO all organization any right to join the Company as a co-
surnamed GUINGON, plaintiffs-appellees, vs. ILUMINADO DEL defendant in any action against the Insured to determine
MONTE, JULIO AGUILAR and CAPITAL INSURANCE and the Insured's liability.
SURETY CO., INC., defendants.
During the effectivity of such policy, del Monte, one of the drivers
G.R. No. L-22042 August 17, 1967 of the jeepneys operated by Aguilar, bumped and killed Guingon.
Homicide thru reckless imprudence was filed against Iluminado del
BENGZON, J.P., J.: Monte, who pleaded guilty.

FACTS: Julio Aguilar owned and operated several jeepneys in As a corollary to such action, the heirs of Gervacio Guingon filed an
the City of Manila. He entered into a contract with the Capital action for damages praying that the sum of P82,771.80 be paid to
Insurance & Surety Co., Inc. insuring the operation of his jeepneys them jointly and severally by the defendants, driver Iluminado del
against accidents with third-party liability. An insurance policy was Monte, owner and operator Julio Aguilar, and the Capital Insurance
then executed. The pertinent provisions of which: & Surety Co., Inc. For failure to answer the complaint, Del Monte
and Aguilar were declared in default. Capital Insurance & Surety
Co., Inc. answered, alleging that the plaintiff has no cause of action
Section II LIABILITY TO THE PUBLIC against it. xxx Appellant contends that the "no action" clause in the
policy closes the avenue to any third party which may be injured in
an accident wherein the jeepney of the insured might have been MORELAND, J.:
the cause of the injury of third persons, alleging the freedom of
contracts. FACTS: The plaintiffs and defendant are brother and sisters. They
are the only heirs at law and next of kin of Gregorio Nacianceno del
ISSUES: (1) Can the plaintiffs sue the insurer. (cause of Val who died intestate. Gregorio took out insurance on his life for
action) the sum of P40,000 and made it payable to the defendant as sole
(2) If so, can plaintiffs sue the insurer jointly with the beneficiary.
insured.
After his death the defendant collected the face of the policy, paid
RULING: YES (to both). The policy in the present case, as the sum of P18,365.20 to redeem certain real estate which the
aforequoted, is one whereby the insurer agreed to indemnify the decedent had sold to third persons with a right to repurchase and
insured "against all sums . . . which the Insured shall become took possession of most of his personal property and the balance
legally liable to pay in respect of: a. death of or bodily injury to any on said insurance policy amounting to P21,634.80.
person . . . ." Clearly, therefore, it is one for indemnity against
liability;1 from the fact then that the insured is liable to the third Plaintiffs contend that the amount of the insurance policy belonged
person, such third person is entitled to sue the insurer.1wph1.t to the estate of the deceased and not to the defendant personally;
that, therefore, they are entitled to a partition not only of the real
The right of the person injured to sue the insurer of the party at and personal property, but also of the P40,000 life insurance. The
fault (insured), depends on whether the contract of insurance is complaint prays a partition of all the property, both real and
intended to benefit third persons also or only the insured. And the personal, left by the deceased; that the defendant account for
test applied has been this: Where the contract provides for P21,634.80, and that that sum be divided equally among the
indemnity against liability to third persons, then third persons to plaintiffs and defendant along with the other property of deceased.
whom the insured is liable, can sue the insurer. Where the contract
is for indemnity against actual loss or payment, then third persons The defendant denies the material allegations of the complaint and
cannot proceed against the insurer, the contract being solely to sets up as special defense and counterclaim that the redemption of
reimburse the insured for liability actually discharged by him thru the real estate sold by his father was made in the name of the
payment to third persons, said third persons' recourse being thus plaintiffs and himself instead of in his name alone without his
limited to the insured alone. knowledge or consent; and that it was not his intention to use the
proceeds of the insurance policy for the benefit of any person but
The "no action" clause in the policy of insurance cannot prevail himself, he alleging that he was and is the sole owner thereof and
over the Rules of Court provision aimed at avoiding multiplicity of that it is his individual property. He, therefore, asks that he be
suits. xxx Similarly, in the instant suit, Sec. 5 of Rule 2 on "Joinder declared the owner of the real estate redeemed by the payment of
of causes of action" and Sec. 6 of Rule 3 on "Permissive joinder of the P18,365.20, the owner of the remaining P21,634.80, the
parties" cannot be superseded, at least with respect to third balance of the insurance policy, and that the plaintiff's account for
persons not a party to the contract, as herein, by a "no action" the use and occupation of the premises so redeemed since the date
clause in the contract of insurance. of the redemption.

Issue: Can the proceeds of the policy be divided among the heirs?

G.R. No. L-9374 February 16, 1915 HELD: YES

FRANCISCO DEL VAL, ET AL., plaintiffs-appellants, vs. With the finding of the trial court that the proceeds of the life-
ANDRES DEL VAL, defendant-appellee. insurance policy belong exclusively to the defendant as his
individual and separate property, we agree. That the proceeds of
an insurance policy belong exclusively to the beneficiary and not to the repurchase of the real estate sold by the decedent prior to his
the estate of the person whose life was insured, and that such death with right to repurchase, and such repurchase having been
proceeds are the separate and individual property of the made and the conveyance taken in the names of all of the heirs
beneficiary, and not of the heirs of the person whose life was instead of the defendant alone, plaintiffs claim that the property
insured, is the doctrine in America. We believe that the same belongs to the heirs in common and not to the defendant alone.
doctrine obtains in these Islands by virtue of section 428 of the
Code of Commerce, which reads: We are not inclined to agree with this contention unless the fact
appear or be shown that the defendant acted as he did with the
The amount which the underwriter must deliver to the intention that the other heirs should enjoy with him the ownership
person insured, in fulfillment of the contract, shall be the of the estate in other words, that he proposed, in effect, to make
property of the latter, even against the claims of the a gift of the real estate to the other heirs. If it is established by the
legitimate heirs or creditors of any kind whatsoever of the evidence that that was his intention and that the real estate was
person who effected the insurance in favor of the former. delivered to the plaintiffs with that understanding, then it is
probable that their contention is correct and that they are entitled
It is claimed by the attorney for the plaintiffs that the section just to share equally with the defendant therein. If, however, it appears
quoted is subordinate to the provisions of the Civil Code as found in from the evidence in the case that the conveyances were taken in
article 1035. This article reads: the name of the plaintiffs without his knowledge or consent, or that
it was not his intention to make a gift to them of the real estate,
An heir by force of law surviving with others of the same then it belongs to him. If that facts are as stated, he has two
character to a succession must bring into the hereditary remedies. The one is to compel the plaintiffs to reconvey to him
estate the property or securities he may have received from and the other is to let the title stand with them and to recover from
the deceased during the life of the same, by way of dowry, them the sum he paid on their behalf.
gift, or for any good consideration, in order to compute it in
fixing the legal portions and in the account of the division. THE INSULAR LIFE ASSURANCE COMPANY, LTD., plaintiff-
appellee,
Counsel also claim that the proceeds of the insurance policy were a vs.
donation or gift made by the father during his lifetime to the CARPONIA T. EBRADO and PASCUALA VDA. DE
defendant and that, as such, its ultimate destination is determined EBRADO, defendants-appellants.
by those provisions of the Civil Code which relate to donations,
especially article 819. This article provides that "gifts made to G.R. No. L-44059. October 28, 1977
children which are not betterments shall be considered as part of
their legal portion." Facts:

We cannot agree with these contentions. The contract of life Buenaventura Ebrado was insured by The Life Assurance Co., Ltd.
insurance is a special contract and the destination of the proceeds on a whole-life insurance policy for P5,882.00 with a rider for
thereof is determined by special laws which deal exclusively with Accidental Death for the same amount. Buenaventura designated
that subject. The Civil Code has no provisions which relate directly his common-law wife, Carponia Ebrado, as the revocable
and specifically to life- insurance contracts or to the destination of beneficiary in his policy. When Buenaventura died as a result of an
life insurance proceeds. That subject is regulated exclusively by the accident when he was hit by a falling branch of a tree, Carponia
Code of Commerce which provides for the terms of the contract, filed with the insurer a claim for the proceeds of the Policy as the
the relations of the parties and the destination of the proceeds of designated beneficiary therein, although she admits that she and
the policy. the insured Buenaventura were merely living as husband and wife2
without the benefit of marriage. Pascuala Vda. de Ebrado also filed
The proceeds of the life-insurance policy being the exclusive her claim as the widow of the deceased insured. She asserts that
property of the defendant and he having used a portion thereof in
she is the one entitled to the insurance proceeds, not the common- The following donations shall be void:
law wife. 1. Those made between persons who were guilty of adultery or
concubinage at the time of donation;
In doubt as to whom the insurance proceeds shall be paid, the Those made between persons found guilty of the same criminal
insurer commenced an action for interpleader. During the pre-trial offense, in consideration thereof;
conference, Carponia and Pascuala agreed and stipulated, among 3. Those made to a public officer or his wife, descendants or
others that: 1) that the deceased Buenaventura was married to ascendants by reason of his office.
Pascuala with whom she has 6 legitimate children and 2) that In the case referred to in No. 1, the action for declaration of nullity
during the lifetime of Buenaventura, he was living with his may be brought by the spouse of the donor or donee; and the guilt
common-wife, Carponia, with whom she had 2 children although he of the donee may be proved by preponderance of evidence in the
was not legally separated from his legal wife. same action.

The trial court rendered judgment declaring Carponia disqualified (2) In essence, a life insurance policy is no different from a civil
from becoming beneficiary of the insured Buenaventura and donation insofar as the beneficiary is concerned. Both are founded
directing the payment of the insurance proceeds to the estate of upon the same consideration: liberality. A beneficiary is like a
the deceased insured. The trial court held: It is patent from the last donee, because from the premiums of the policy which the insured
paragraph of Art. 739 of the Civil Code that a criminal conviction for pays out of liberality, the beneficiary will receive the proceeds or
adultery or concubinage is not essential in order to establish the profits of said insurance. As a consequence, the proscription in
disqualification mentioned therein. Neither is it also necessary that Article 739 of the new Civil Code should equally operate in life
a finding of such guilt or commission of those acts be made in a insurance contracts. The mandate of Article 2012 cannot be laid
separate independent action brought for the purpose. The guilt of aside: any person who cannot receive a donation cannot be named
the donee (beneficiary) may be proved by preponderance of as beneficiary in the life insurance policy of the person who cannot
evidence in the same proceeding (the action brought to declare the make the donation.
nullity of the donation).

Issue:
Whether or not a common-law wife, named as beneficiary in the [G.R. Nos. 128833. April 20, 1998]
life insurance policy of a legally married man, can claim the
proceeds thereof in case of death of the latter. RIZAL COMMERCIAL BANKING CORPORATION, UY CHUN BING AND
ELI D. LAO, petitioners, vs. COURT OF APPEALS and GOYU & SONS,
Ruling: INC., respondents.
NO, the common-law wife cannot claim the insurance proceeds. Facts:
(1) It is quite unfortunate that the Insurance Act (RA 2327, as RCBC Binondo Branch initially granted a credit facility of P30M to
amended) or even the new Insurance Code (PD No. 612, as Goyu & Sons, Inc. GOYUs applied again and through Binondo
amended) does not contain any specific provision grossly resolutory Branch key officer's Uys and Laos recommendation, RCBCs
of the prime question at hand. executive committee increased its credit facility to P50M to P90M
Rather, the general rules of civil law should be applied to resolve and finally to P117M.As security, GOYU executed 2 real estate
this void in the Insurance Law. Article 2011 of the New Civil Code mortgages and 2 chattel mortgages in favor of RCBC. GOYU
states: "The contract of insurance is governed by special laws. obtained in its name 10 insurance policy on the mortgaged
Matters not expressly provided for in such special laws shall be properties from Malayan Insurance Company, Inc. (MICO), sister
regulated by this Code." And under Article 2012 of the same Code, company of RCBC. In February 1992, he was issued 8 insurance
"any person who is forbidden from receiving any donation under policies in favor of RCBC.
Article 739 cannot be named beneficiary of a fife insurance policy April 27, 1992: One of GOYUs factory buildings was burned so he
by the person who cannot make a donation to him. Common-law claimed against MICO for the loss who denied contending that the
spouses are, definitely, barred from receiving donations from each insurance policies were either attached pursuant to writs of
other. Article 739 of the new Civil Code provides:
attachments/garnishments or that creditors are claiming to have a been specifically disclosed by the insured itself. It is also significant
better right that GOYU voluntarily and purposely took the insurance policies
GOYU filed a complaint for specific performance and damages at from MICO, a sister company of RCBC, and not just from any other
the RTC insurance company. Alchester would not have found out that the
RCBC, one of GOYUs creditors, also filed with MICO its formal claim subject pieces of property were mortgaged to RCBC had not such
over the proceeds of the insurance policies, but said claims were information been voluntarily disclosed by GOYU itself. Had it not
also denied for the same reasons that MICO denied GOYUs claims been for GOYU, Alchester would not have known of GOYU's
RTC: Confirmed GOYUs other creditors (Urban Bank, Alfredo intention of obtaining insurance coverage in compliance with its
Sebastian, and Philippine Trust Company) obtained their writs of undertaking in the mortgage contracts with RCBC, and verify,
attachment covering an aggregate amount of P14,938,080.23 and Alchester would not have endorsed the policies to RCBC had it not
ordered that 10 insurance policies be deposited with the court been so directed by GOYU.
minus the said amount so MICO deposited P50,505,594.60. Another
Garnishment of P8, 696,838.75 was handed down To permit GOYU to capitalize on its non-confirmation of these
RTC: favoured GOYU against MICO for the claim, RCBC for damages endorsements while it continued to enjoy the benefits of the credit
and to pay RCBC its loan facilities of RCBC which believed in good faith that there was due
CA: Modified by increasing the damages in favor of GOYU endorsement pursuant to their mortgage contracts, is to
In G.R. No. 128834, RCBC seeks right to intervene in the action countenance grave contravention of public policy, fair dealing,
between Alfredo C. Sebastian (the creditor) and GOYU (the debtor), good faith, and justice. Such an unjust situation, the Court cannot
where the subject insurance policies were attached in favor of sanction. Under the peculiar circumstances obtaining in this case,
Sebastian the Court is bound to recognize RCBC's right to the proceeds of the
RTC and CA: endorsements do not bear the signature of any officer insurance policies if not for the actual endorsement of the policies,
of GOYU concluded that the endorsements favoring RCBC as at least on the basis of the equitable principle of estoppel.
defective.
ISSUE: W/N RCBC as mortgagee, has any right over the insurance
policies taken by GOYU, the mortgagor, in case of the occurrence of
loss GOYU cannot seek relief under Section 53 of the Insurance Code
Held: RCBC has a right over the insurance proceeds. which provides that the proceeds of insurance shall exclusively
apply to the interest of the person in whose name or for whose
It is settled that a mortgagor and a mortgagee have separate and benefit it is made. The peculiarity of the circumstances obtaining in
distinct insurable interests in the same mortgaged property, such the instant case presents a justification to take exception to the
that each one of them may insure the same property for his own strict application of said provision, it having been sufficiently
sole benefit. There is no question that GOYU could insure the established that it was the intention of the parties to designate
mortgaged property for its own exclusive benefit. In the present RCBC as the party for whose benefit the insurance policies were
case, although it appears that GOYU obtained the subject insurance taken out. Consider thus the following:
policies naming itself as the sole payee, the intentions of the
parties as shown by their contemporaneous acts, must be given 1. It is undisputed that the insured pieces of property were the
due consideration in order to better serve the interest of justice and subject of mortgage contracts entered into between RCBC and
equity. GOYU in consideration of and for securing GOYU's credit facilities
from RCBC. The mortgage contracts contained common provisions
whereby GOYU, as mortgagor, undertook to have the mortgaged
property properly covered against any loss by an insurance
It is to be noted that 9 endorsement documents were prepared by company acceptable to RCBC.
Alchester in favor of RCBC. The Court is in a quandary how
Alchester could arrive at the idea of endorsing any specific 2. GOYU voluntarily procured insurance policies to cover the
insurance policy in favor of any particular beneficiary or payee mortgaged property from MICO, no less than a sister company of
other than the insured had not such named payee or beneficiary RCBC and definitely an acceptable insurance company to RCBC.
He paid the agent P15 as part of the first premium. It was
3. Endorsement documents were prepared by MICO's agreed that the policy, when and if issued, should be
underwriter, Alchester Insurance Agency, Inc., and copies thereof delivered to Felicidad with whom Sindayen left the sum
were sent to GOYU, MICO and RCBC. GOYU did not assail, until of P25.06 to complete the payment of the first annual premium
late, the validity of said endorsements. of P40.06
On Jan 1, 1933, Sindayen was examined by Insulars doctor
4. GOYU continued until the occurrence of the fire, to enjoy the who made a favorable report to Insular.
benefits of the credit facilities extended by RCBC which was The next day, Sindayen returned to Manila and resumed his
conditioned upon the endorsement of the insurance policies to be work. On Jan. 11, 1933, Insular accepted the risk and issued
taken by GOYU to cover the mortgaged properties. a policy, and mailed the same to its agent for delivery to the
insured.
On Jan. 12, 1933, Sindayen complained of a severe
headache. ON Jan. 15, 1933, he called a physician who
This Court cannot over stress the fact that upon receiving its copies
found that Sindayen was suffering from acute nephritis and
of the endorsement documents prepared by Alchester, GOYU,
uremia. His illness did not yield to treatment and on Jan. 19,
despite the absence written conformity thereto, obviously
1933, he died
considered said endorsement to be sufficient compliance with its
The policy which the company issued and mailed in manila
obligation under the mortgage contracts since RCBC accordingly
continued to extend the benefits of its credit facilities and GOYU on Jan. 11 1933 was received by its agent in Camilin on Jan.
continued to benefit therefrom. Just as plain too is the intention of 16, 1933. On Jan 18, 1933, the agent, in accordance with his
the parties to constitute RCBC as the beneficiary of the various agreement with the insured delivered the policy to Felicided
insurance policies obtained by GOYU. The intention of the parties upon her payment of the balance of the 1st years premium.
will have to be given full force and effect in this particular case. The The agent asked Felicidad if her nephew was in good health
insurance proceeds may, therefore, be exclusively applied to RCBC, and she replied that she believed so because she had no
which under the factual circumstances of the case, is truly the information that he was sick, and thereupon , the policy was
person or entity for whose benefit the policies were clearly handed to her by the agent.
intended. On Jan. 20, 1933, the agent learned of the death of
Sindayen, afterwhich he called upon Felicidad and asked her
SINDAYEN V INSULAR LIFE to return the policy. Felicidad did so.
On Feb. 4, 1933, the company obtained from Sindayens
Doctrine: widow Fortunata (also the beneficiary), her signature on a
The delivery of the policy to the insured by an agent of the legal document whereby in consideration of the sum 40.06
company who is authorized to make delivery or withhold representing the amount of premium paid, Fortunata
delivery is the final act which binds the company and the thereby releases forever and discharges Insular from any
insured, in the absence of fraud or other legal grounds for and all claims and obligations she may have against the
rescission latter.
A check for the above-mentioned amount was drawn in the
FACTS: name of Fortunata, but the same was never encashed.
Arturo Sindayen was a linotype operator in the Bureau of Instead, it was returned to Insular and this complaint to
Printing. He and his wife Fortunata went to Tarlac to spend enforce payment under the policy was instituted.
Christmas with his aunt Felicidad Estrada. The application which Sindayen signed in Camiling
On Dec. 26, 1932, while still in Camiling, he made a written contained the following provisions:
application to Insular Life, through its agent, Cristobal (3) That the said policy shall not take effect until the first
Hendoza, for a policy of insurance on his life in the sum of premium has been paid and the policy has been delivered
1,000 to and accepted by me, while I am in good health.
The main defense of the company is the policy never took without risk of either loss or of litigation is the bedrock of life
effect because of par. 3 of the application, since at the time insurance.
of the delivery of the agent, the insured was not in good A cloud will be thrown over the entire insurance business if
health. the condition of health of the insured at the time of the
delivery of the policy may be inquired into years afterwards
ISSUE: WON the policy already took effect. with the view of avoiding the policy on the ground that it
never took effect because of an alleged lack of good health
HELD: at the time of delivery.
YES That the delivery of the policy to the insured by an agent of
An agent to whom a life insurance policy (similar to the one the company who is authorized to make delivery or withhold
at bar) was sent with instruction to deliver it to the insured, delivery is the final act which binds the company and the
has authority to bind the company by making such delivery, insured, in the absence of fraud or other legal grounds for
ALTHOUGH the insured was NOT in good health at the time rescission. The fact that the agent to whom it has entrusted
of delivery, on the theory that the delivery of the policy this duty is derelict or negligent or even dishonest in the
being the final act to the consummation of the contract, the performance of the duty which has been entrusted to him
condition as to the insureds good health was WAIVED by the would create an obligation based upon the authorized acts
company. of the agent toward a third party who was not in collusion
That the delivery of the policy by the agent to the insured with the agent.
consummates the contract even though the agent knew that
the insured was NOT in good health at the time, the theory
being, that his knowledge is the companys knowledge; and G.R. No. L-5915 March 31, 1955
his delivery is the companys delivery; that when the
delivery is made notwithstanding this knowledge of the EAGLE STAR INSURANCE CO., LTD., KURR STEAMSHIP CO.,
defect, the company is deemed to have WAIVED such INC., ROOSEVELT STEAMSHIP AGENCY, INC., and LEIF HOEGH
defect. & COMPANY, A/S., petitioners,
The agent, Mendoza was duly licensed by the Insurance vs.
Commission to act for Insular Life. He had the authority CHIA YU, respondent.
given by him by the company to withhold the delivery of the
policy to the insured until the first premium has been paid Reyes, A. J.
and the policy has been delivered to and accepted by the
insured while he is in good health. Whether that condition
Facts:
had been met or not plainly calls for the exercise of
discretion. Mendozas decision that the condition had been
met by the insured and that it was proper to make delivery Atkin, Kroll & Co., loaded 14 bales of assorted underwear valued at
of the policy to him is just as binding on the company as if P8,085.23 consigned to Chia Yu in the City of Manila. The shipment
the decision had been made by its Board of Directors. was insured against all risks by Eagle Star Ins. Co. of San Francisco,
Admittedly, Mendoza made a mistake of judgment because California, under a policy issued to the shipper and by the latter
he acted on insufficient evidence as to the state of health of assigned to the consignee. The vessel arrived in Manila and started
the insured, and this mistake cannot be said to be induced discharging its cargo into the custody of an arrastre operator .
by any misconduct on the part of the insured. Later, 10 out of 14 bales consigned to Chia Yu were only delivered.
It is in the interest of not only of the applicant but of all
insurance companies as well that there should be some act Chia Yu claimed indemnity for the missing and damaged bales. But
which gives the applicant the definite assurance that the the claim was declined, first, by the carrier and afterward by the
contract has been consummated. This sense of security and insurer, whereupon Chia Yu brought the present action against
of peace of mind that ones dependents are provided for both, including their respective agents in the Philippines.
More than two years after delivery of the damaged bales and the his action to less than one year. This is so because the said clause
date when the missing bales should have been delivered, the makes the prescriptive period begin from the happening of the loss
action was resisted by the insurer principally on the ground of and at the same time provides that the no suit on the policy shall
prescription. The trial court found for Chia Yu and rendered be sustainable in any court unless the insured shall have first fully
judgment in his favor for the sum claimed plus legal interest and complied with all the terms and conditions of the policy, among
costs. The judgment was affirmed by the Court of Appeals, and the them that which requires that, as so as the loss is determined,
case is now before us on appeal by certiorari. written claim therefor be filed with the carrier and that the letter to
the carrier and the latter's reply should be attached to the claim
On the part of the carrier the defense of prescription is made to papers to be sent to the insurer. It is obvious that compliance with
rest on the following stipulation of the bill of lading: In any event this condition precedent will necessarily consume time and thus
the carrier and the ship shall be discharged from all liability in shorten the period for bringing suit to less than one year if the
respect of loss or damage unless suit is brought within one year period is to begin, as stated in the policy, from "the happening of
after the delivery of the goods or the date when the goods should the loss." Being contrary to the law of the forum, such stipulation
have been delivered. cannot be given effect.

The case for the insurer stands on a different footing, for its claim It may perhaps be suggested that the policy clause relied on by the
of prescription is founded upon the terms of the policy and not insurer for defeating plaintiff's action should be given the
upon the bill of lading: No suit action on this Policy, for the construction that would harmonize it with section 61-A of the
recovery of any claim, shall be sustainable in any Court of law or Insurance Act by taking it to mean that the time given the insured
equity unless the insured shall have fully complied with all the for bringing his suit is twelve months after the cause of action
terms and conditions of this Policy nor unless commenced with accrues. But the question then would be: When did the cause of
twelve (12) months next after the happening of the loss action accrue? On that question we agree with the court below that
plaintiff's cause of action did not accrue until his claim was finally
Issue rejected by the insurance company. This is because, before such
final rejection, there was no real necessity for bringing suit. As the
policy provides that the insured should file his claim, first, with the
Whether the plaintiff's action against the insurer has prescribed? carrier and then with the insurer, he had a right to wait for his claim
to be finally decided before going to court. The law does not
Held: encourages unnecessary litigation.

No. Any condition, stipulation or agreement in any policy of Sun Insurance Office, Ltd. v. CA and Emilio Tan
insurance, limiting the time for commencing an action thereunder G.R. No. 89741 March 13, 1991
to a period of less than one year from the time when the cause of Paras, J.
action accrues, is void. FACTS:
Emilio Tan took from Sun Insurance Office a P300,000.00 property
As "matters respecting a remedy, such as the bringing of suit, insurance policy tocover his interest in the electrical supply store of
admissibility of evidence, and statute of limitations, depend upon his brother. Four days after the issuance of the policy, the building
the law of the place where the suit is brought" (Insular was burned including the insured store. On August 20, 1983,
Government vs. Frank, 13 Phil. 236), any policy clause repugnant to Tanfiled his claim for fire loss with Sun Insurance Office, but on
this amendment to the Insurance Act cannot be given effect in an February 29, 1984, SunInsurance Office wrote Tan denying the
action in our courts. latters claim. On April 3, 1984, Tan wrote SunInsurance Office,
seeking reconsideration of the denial of his claim. Sun Insurance
Examining the policy sued upon in the present case, we find that its Officeanswered the letter, advising Tans counsel that the Insurers
prescriptive clause, if given effect in accordance with the terms of denial of Tans claim remainedunchanged.
the policy, would reduce the period allowed the insured for bringing ISSUES:
(1)WON the filing of a motion for reconsideration interrupts the 12 mere filing of a plea for reconsideration of a denial is sufficient
monthsprescriptive period to contest the denial of the insurance ormust it be supported by arguments/affidavits/material
claim; and(2)WON the rejection of the claim shall be deemed final evidence;b.how many petitions for reconsideration should be
only of it contains words tothe effect that the denial is final; permitted?(2) No. The Eagle Star case cited by Tan to defend his
HELD: theory that the rejection of theclaim shall be deemed final only of it
(1) No. In this case, Condition 27 of the Insurance Policy of the contains words to the effect that the denial is final isinapplicable in
parties reads:27. the instant case. Final rejection or denial cannot be taken to mean
Action or suit clause therejection of a petition for reconsideration. The Insurance policy
- If a claim be made and rejected and anaction or suit be not in theEagle Star case provides that the insured should file his claim,
commenced either in the Insurance Commission orin any court of first, with the carrier and then with theinsurer. The final rejection
competent jurisdiction within twelve (12) months fromreceipt of being referred to in said case is the rejection by the
notice of such rejection, or in case of arbitration taking placeas insurancecompany.
provided herein, within twelve (12) months after due notice of
theaward made by the arbitrator or arbitrators or umpire, then the
claimshall for all purposes be deemed to have been abandoned and
shallnot thereafter be recoverable hereunder.As the terms are very G.R. No. L-15862 July 31, 1961
clear and free from any doubt or ambiguity whatsoever, it mustbe
taken and understood in its plain, ordinary and popular sense. Tan,
in his letter addressed to Sun Insurance Office dated April 3, 1984, PAULO ANG and SALLY C. ANG vs. FULTON FIRE INSURANCE
admitted thathe received a copy of the letter of rejection on April 2, CO., ET AL.,
1984. Thus, the 12-monthprescriptive period started to run from
the said date of April 2, 1984, for such is the plainmeaning and Facts: On September 9, 1953, Fulton Fire Insurance Company
intention of Section 27 of the insurance policy. The condition issued a policy in favor of P. & S Department Store (Sally C. Ang)
contained in an insurance policy that claims must be presented over stocks of general merchandise, consisting principally of dry
withinone year after rejection is not merely a procedural goods, contained in a building occupied by the plaintiffs. The
requirement but an important matteressential to a prompt premium is P500.00 annually. The insurance was issued for one
settlement of claims against insurance companies as it demands year, but the same was renewed for another year on September
thatinsurance suits be brought by the insured while the evidence as 31, 1954. On December 17, 1954, the store containing the goods
to the origin and cause of destruction have not yet disappeared.It is insured was destroyed by fire. On December 30, plaintiffs executed
apparent that Section 27 of the insurance policy was stipulated the first claim form. The claim together with all the necessary
pursuant toSection 63 of the Insurance Code, which states that:Sec. papers relating thereto, were forwarded to the Manila Adjustment
63. A condition, stipulation or agreement in any policy of insurance, Company, the defendants' adjusters and received by the latter on
limiting the time for commencing an action thereunder to a period June 8, 1955. On January 12, 1955, the Manila Adjustment
of less than one year from the time when the cause of Company accepted receipt of the claim and requested the
actionaccrues, is void.It also begs to ask, when does the cause of submission of the books of accounts of the insured for the year
action accrue? The insureds cause of action or his right to file a 1953-1954 and a clearance from the Philippine Constabulary and
claim either in the Insurance Commission or in a court of the police. On April 6, 1956, the Fulton Fire Insurance Company
competent jurisdiction commences from the time of the denial of wrote the plaintiffs that their claim was denied. This denial of the
his claim by the Insurer,either expressly or impliedly. But the claim was received by the plaintiffs on April 19, 1956. On January
rejection referred to should be construed as therejection in the first 13, 1955, Paulo Ang and ten others were charged for arson. The
instance (i.e. at the first occasion or for the first time), not case was remanded for trial to the Court of First Instance of Ilocos
rejectionconveyed in a resolution of a petition for reconsideration. Norte which acquitted Paulo Ang.
Thus, to allow the filing of amotion for reconsideration to suspend
the running of the prescriptive period of twelvemonths, a whole On May 26, 1958, the defendant Fulton Fire Insurance Company
new body of rules on the matter should be promulgated so as to filed an answer to the complaint, admitting the existence of the
avoid anyconflict that may be brought by it, such as:a.whether the contract of insurance, its renewal and the loss by fire of the
department store and the merchandise contained therein, but terms. Actually, what was imported was 59.940 metric tons
denying that the loss by the fire was accidental, alleging that it was not 600 tons at $395.42 a ton CNF Manila. The fishmeal in
occasioned by the willful act of the plaintiff Paulo Ang himself. It 666 new gunny bags were unloaded from the ship on
claims that under paragraph 13 of the policy, if the loss or damage December 11, 1976 at Manila unto the arrastre contractor E.
is occasioned by the willful act of the insured, or if the claim is Razon, Inc. and defendant's surveyor ascertained and
made and rejected but no action is commenced within 12 months certified that in such discharge 105 bags were in bad order
after such rejection, all benefits under the policy would be forfeited, condition as jointly surveyed by the ship's agent and the
and that since the claim of the plaintiffs was denied and plaintiffs arrastre contractor. It was later found out that the extent of
received notice of denial on April 18, 1956, and they brought the shortage or loss on the bad order bags totalling 227 bags
action only on May 5, 1958, all the benefits under the policy have amounting to 12,148 kilos. Based on said computation the
been forfeited. plaintiff made a formal claim against the defendant Filipino
Merchants Insurance Company for P51,568.62, but the
Issue: WON the action already prescribes? defendant Filipino Merchants Insurance Company refused to
pay the claim. Consequently, the plaintiff brought an action
Held: Yes. The condition contained in the insurance policy that against said defendant as adverted to above and defendant
claims must be presented within one year after rejection is not presented a third party complaint against the vessel and the
merely a procedural requirement. The condition is an important arrastre contractor.
matter, essential to a prompt settlement of claims against
insurance companies, as it demands that insurance suits be RTC Manila rendered judgment in favor of private
brought by the insured while the evidence as to the origin and respondent
cause of destruction have not yet disappeared. It is in the nature of
a condition precedent to the liability of the insurer, or in other CA affirmed the decision of the lower court
terms, a resolutory cause, the purpose of which is to terminate all
liabilities in case the action is not filed by the insured within the ISSUE: Whether Filipino Merchant may be held liable under
period stipulated. the all risks clause of the marine insurance policy in the
absence of proof of some fortuitous event, casualty, or
accidental cause to which the loss is attributable.
Filipino Merchants Insurance Co Inc vs. CA HELD:
G.R. No. 85141. November 28, 1989 The SC ruled that Filipino Merchant is liable under the
all risks clause of the marine insurance policy it had with
Choa.
FACTS:
This is an action brought by the consignee of the Filipino Merchant contended that an "all risks" marine
shipment of fishmeal loaded on board the vessel SS policy has a technical meaning in insurance in that before a
Bougainville and unloaded at the Port of Manila on or about claim can be compensable it is essential that there must be
December 11, 1976 and seeks to recover from the "some fortuity, " "casualty" or "accidental cause" to which
defendant insurance company the amount of P51,568.62 the alleged loss is attributable and the failure of herein
representing damages to said shipment which has been private respondent, upon whom lay the burden, to adduce
insured by Filipino Merchant. evidence showing that the alleged loss to the cargo in
question was due to a fortuitous event precludes his right to
Plaintiff insured said shipment with defendant recover from the insurance policy.
insurance company for the sum of P267,653.59 for the
goods described as 600 metric tons of fishmeal in new The SC ruled in the negative. It explained that an "all
gunny bags of 90 kilos each from Bangkok, Thailand to risks policy" should be read literally as meaning all risks
Manila against all risks under warehouse to warehouse whatsoever and covering all losses by an accidental cause
of any kind. The terms "accident" and "accidental", as used APPEALS, SOUTH SEA SURETY AND INSURANCE CO., INC.
in insurance contracts, have not acquired any technical and the CHARTER INSURANCE CORPORATION
meaning. They are construed by the courts in their ordinary G.R. No. 124050. June 19, 1997
and common acceptance. The term "all risks" cannot be PUNO, J.
given a strained technical meaning, the language of the FACTS: Petitioner Hongkong contracted petitioner Mayer to
clause under the Institute Cargo Clauses being unequivocal manufacture and supply various steel pipes and fittings. From
and clear, to the effect that it extends to all damages/losses August to October, 1983, Mayer shipped the pipes and fittings to
suffered by the insured cargo except (a) loss or damage or Hongkong. Prior to the shipping, Mayer insured the pipes and
expense proximately caused by delay, and (b) loss or fittings against all risks with private respondents South Sea Surety
damage or expense proximately caused by the inherent vice and Insurance Co., Inc. (South Sea) and Charter Insurance Corp.
or nature of the subject matter insured. Generally, the (Charter).
burden of proof is upon the insured to show that a loss arose Industrial Inspection, a 3rd party inspector, certified all the pipes
from a covered peril, but under an "all risks" policy the and fittings to be in good order condition before they were loaded
burden is not on the insured to prove the precise cause of in the vessel. Nonetheless, when the goods reached Hongkong, it
loss or damage for which it seeks compensation. The was discovered that a substantial portion thereof was damaged.
insured under an "all risks insurance policy" has the initial Petitioners filed a claim against private respondents for indemnity
burden of proving that the cargo was in good condition under the insurance contract. Respondent Charter paid petitioner
when the policy attached and that the cargo was damaged Hongkong the amount of HK$64,904.75. Petitioners demanded
when unloaded from the vessel; thereafter, the burden then payment of the balance of HK$299,345.30 representing the cost of
shifts to the insurer to show the exception to the coverage. repair of the damaged pipes. Private respondents refused to pay
because the insurance surveyor's report allegedly showed that the
In the present case, there being no showing that the damage is a factory defect which are not covered by the insurance
loss was caused by any of the excepted perils, the insurer is policies.
liable under the policy. There is no evidence presented to Trial court ruled in favor of petitioners. On appeal, CA affirmed the
show that the condition of the gunny bags in which the finding of the trial court that the damage is not due to factory
fishmeal was packed was such that they could not hold their defect and that it was covered by the "all risks" insurance policies
contents in the course of the necessary transit, much less issued by private respondents to petitioner Mayer. However, it set
any evidence that the bags of cargo had burst as the result aside the decision of the trial court and dismissed the complaint on
of the weakness of the bags themselves. Had there been the ground of prescription. It held that the action is barred under
such a showing that spillage would have been a certainty, Section 3(6) of the Carriage of Goods by Sea Act since it was filed
there may have been good reason to plead that there was only on April 17, 1986, more than two years from the time the
no risk covered by the policy. goods were unloaded from the vessel. Section 3(6) of the Carriage
of Goods by Sea Act provides that "the carrier and the ship shall be
NOTE: discharged from all liability in respect of loss or damage unless suit
C & F contracts are shipment contracts. The term is brought within one year after delivery of the goods or the date
means that the price fixed includes in a lump sum the cost when the goods should have been delivered." Respondent court
of the goods and freight to the named destination. 21 It ruled that this provision applies not only to the carrier but also to
simply means that the seller must pay the costs and freight the insurer, citing Filipino Merchants Insurance Co., Inc. vs.
necessary to bring the goods to the named destination but Alejandro.
the risk of loss or damage to the goods is transferred from ISSUE: Whether or not 3(6) of COGSA also applies to the
the seller to the buyer when the goods pass the ship's rail in insurer.
the port of shipment. RULING: NO, respondent court erred in applying Section
3(6) of COGSA. Under this provision, only the carrier's liability is
MAYER STEEL PIPE CORPORATION and HONGKONG extinguished if no suit is brought within one year. But the liability
GOVERNMENT SUPPLIES DEPARTMENT vs. COURT OF of the insurer is not extinguished because the insurer's liability is
based not on the contract of carriage but on the contract of
insurance. A close reading of the law reveals that the Carriage of taken out with the Springfield Fire & Marine Insurance Company.
Goods by Sea Act governs the relationship between the carrier on The warehouse was destroyed by fire on January 11, 1928, while
the one hand and the shipper, the consignee and/or the insurer on the policy issued by the latter company was in force.
the other hand. It defines the obligations of the carrier under the
contract of carriage. It does not, however, affect the relationship Predicated on this policy the plaintiff instituted action in the Court
between the shipper and the insurer. The latter case is governed of First Instance of Manila against the defendant to recover a
by the Insurance Code. proportional part of the loss coming to P8,170.59. Four special
The Filipino Merchants case is different from the case at bar. In defenses were interposed on behalf of the insurance company, one
Filipino Merchants, it was the insurer which filed a claim against the being planted on a violation of warranty F fixing the amount of
carrier for reimbursement of the amount it paid to the shipper. In hazardous goods which might be stored in the insured building. The
the case at bar, it was the shipper which filed a claim against the trial judge in his decision found against the insurance company on
insurer. The basis of the shipper's claim is the "all risks" insurance all points, and gave judgment in favor of the plaintiff for the sum of
policies issued by private respondents to petitioner Mayer. xxx P8,188.74. From this judgment the insurance company has
When the court said in Filipino Merchants that Section 3(6) of appealed, and it is to the first and fourth errors assigned that we
COGSA applies to the insurer, it meant that the insurer, like the would address particular attention.
shipper, may no longer file a claim against the carrier beyond the
one-year period provided in the law. But it does not mean that the The description of the risk in this policy is as follows:lawphil.net
shipper may no longer file a claim against the insurer because the
basis of the insurer's liability is the insurance contract. An
insurance contract is a contract whereby one party, for a Ten thousand pesos Philippine Currency. On
consideration known as the premium, agrees to indemnify another general non-hazardous merchandise, chiefly
for loss or damage which he may suffer from a specified peril. An consisting of chucherias, also produce, Cacao, Flour,
"all risks" insurance policy covers all kinds of loss other than those all the property of the Insured, or held by them in
due to willful and fraudulent act of the insured. Thus, when trust, on commission or on joint account with others,
private respondents issued the "all risks" policies to or for which he is responsible, while contained during
petitioner Mayer, they bound themselves to indemnify the the currency of this policy in the godown, situate No.
latter in case of loss or damage to the goods insured. Such 643 Calle Reina Regent. . . .
obligation prescribes in ten years, in accordance with
Article 1144 of the New Civil Code. This policy is subject to the hereon attached
"Ordinary Short Period Rate Scale"Warranties A & F,
G.R. No. L-33637 December 31, 1931 Co-insurances Clause "and Three Fourths Loss
Clause," which are forming part of same. Co-
ANG GIOK CHIP, doing business under the name and style of insurance declared:
Hua Bee Kong Si, plaintiff-appellee,
vs. "P20,000. Sun Insurance Office Ltd. (K & S)."
SPRINGFIELD FIRE & MARINE INSURANCE (Emphasis inserted.) Securely pasted on the left hand
COMPANY, defendant-appellant. margin of the face of the policy are five warranties
and special clauses. One of them is warranty F,
FACTS: specially referred to on the face of the policy, reading
in part as follows:
Ang Giok Chip doing business under the name and style of Hua Bee
Kong Si was formerly the owner of a warehouse situated at No. 643 WARRANTY F
Calle Reina Regente, City of Manila. The contents of the warehouse
were insured with the three insurance companies for the total sum It is hereby declared and agreed that during the
of P60,000. One insurance policy, in the amount of P10,000, was currency of this policy no hazardous goods be stored
in the Building to which this insurance applies or in It is hereby declared and agreed that during the currency of this
any building communicating therewith, provided, policy no hazardous goods be stored in the Building to which this
always, however, that the Insured be permitted to insurance applies or in any building communicating therewith,
stored a small quantity of the hazardous goods provided, always, however, that the Insured be permitted to stored
specified below, but not exceeding in all 3 per cent of a small quantity of the hazardous goods specified below, but not
the total value of the whole of the goods or exceeding in all 3 per cent of the total value of the whole of the
merchandise contained in said warehouse, viz; . . . . goods or merchandise contained in said warehouse, viz; . . . .

The applicable law is found in the Instance Act, Act Also, the court stated a book that said, "any express warranty or
No. 2427, as amended, section 65 reading: condition is always a part of the policy, but, like any other part of
an express contract, may be written in the margin, or contained in
"Every express warranty, made at or before the execution of a proposals or documents expressly referred to in the policy, and so
policy, must be contained in the policy itself, or in another made a part of it."
instrument signed by the insured and referred to in the policy, as
making a part of it." It is well settled that a rider attached to a policy is a part of the
contract, to the same extent and with like effect as it actually
As the Philippine law was taken verbatim from the law of California, embodied therein. In the second place, it is equally well settled that
in accordance with well settled canons of statutory construction, an express warranty must appear upon the face of the policy, or be
the court should follow in fundamental points, at least, the clearly incorporated therein and made a part thereof by explicit
construction placed by California courts on a California law. reference, or by words clearly evidencing such intention.
Unfortunately the researches of counsel reveal no authority coming
from the courts of California which is exactly on all fours with the The court concluded that Warranty F is contained in the policy
case before us. However, there are certain consideration lying at itself, because by the contract of insurance agreed to by the parties
the basis of California law and certain indications in the California it was made to be a part. It wasnt aseparate instrument agreed to
decisions which point the way for the decision in this case by the parties.

ISSUE: Whether a warranty referred to in the policy as forming part The receipt of the policy by the insured without objection binds
of the contract of insurance and in the form of a rider to the him. It was his duty to read the policy and know its terms. He also
insurance policy, is null and void because not complying with the never chose to accept a different policy by considering the earlier
Philippine Insurance Act. one as a mistake. Hence, the rider is valid.

HELD: No. The warranty is valid. Petition dismissed. MALAYAN INSURANCE CO., INC. (MICO), petitioner,
vs.
The Insurance Act, Section 65, taken from California law, states: GREGORIA CRUZ ARNALDO, in her capacity as the
INSURANCE COMMISSIONER, and CORONACION
PINCA, respondents.
"Every express warranty, made at or before the execution of a
policy, must be contained in the policy itself, or in another
instrument signed by the insured and referred to in the policy, as G.R. No. L-67835. October 12, 1987
making a part of it."
Facts:
Warranty F, indemnifying for a value of Php 20,000 and pasted on
the left margin of the policy stated: On June 7, 1981, the petitioner (hereinafter called MICO) issued to
Coronacion Pinca, a Fire Insuance Policy on her property for the
amount of P14,000.00 effective July 22, 1981, until July 22, 1982.
On October 15,1981, MICO allegedly cancelled the policy for non- important because it suggests an understanding between MICO and
payment, of the premium and sent the corresponding notice to the insured that such payment could be made later, as agent Adora
Pinca. On December 24, 1981, payment of the premium for Pinca had assured Pinca. In any event, it is not denied that this payment
was received by Domingo Adora, agent of MICO. On January 15, was actually made by Pinca to Adora, who remitted the same to
1982, Adora remitted this payment to MICO, together with other MICO.
payments.
It is not disputed that the preium was actually paid by Pinca to
On January 18, 1982, Pinca's property was completely burned. On Adora on December 24, 1981, who received it on behalf of MICO, to
February 5, 1982, Pinca's payment was returned by MICO to Adora which it was remitted on January 15, 1982. What is questioned is
on the ground that her policy had been cancelled earlier. But Adora the validity of Pinca's payment and of Adora's authority to receive
refused to accept it. it.

In due time, Pinca made the requisite demands for payment, which MICO's acknowledgment of Adora as its agent defeats its
MICO rejected. She then went to the Insurance Commission. The contention that he was not authorized to receive the premium
Commission sustained Pinca, hence the appeal of MICO. payment on its behalf. It is clearly provided in Section 306 of the
Insurance Code that:
Issues:
SEC. 306. xxx xxx xxx
Whether or not the petition made by MICO was still meritorious
Any insurance company which delivers to an insurance agant or
Whether or not there was a valid cancellation of the insurance insurance broker a policy or contract of insurance shall be demmed
policy to have authorized such agent or broker to receive on its behalf
payment of any premium which is due on such policy or contract of
Ruling: insurance at the time of its issuance or delivery or which becomes
due thereon.
Whichever law is applicable, therefore, the petition can and should
be dismissed for late filing. And it is a well-known principle under the law of agency that:

The petitioner relies heavily on Section 77 of the Insurance Code Payment to an agent having authority to receive or collect payment
providing that: is equivalent to payment to the principal himself; such payment is
complete when the money delivered is into the agent's hands and
is a discharge of the indebtedness owing to the principal.
SEC. 77. An insurer is entitled to payment of the premium as soon
as the thing is exposed to the peril insured against.
Notwithstanding any agreement to the contrary, no policy or There is the petitioner's argument, however, that Adora was not
contract of insurance issued by an insurance company is valid and authorized to accept the premium payment because six months
binding unless and until the premium thereof has been paid, except had elapsed since the issuance by the policy itself. It is argued that
in the case of a life or an industrial life policy whenever the grace this prohibition was binding upon Pinca, who made the payment to
period provision applies. Adora at her own riskl as she was bound to first check his authority
to receive it.
The above provision is not applicable because payment of the
premium was in fact eventually made in this case. Notably, the We do not share MICO's view that there was no existing insurance
premium invoice issued to Pinca at the time of the delivery of the at the time of the loss sustained by Pinca because her policy never
policy on June 7, 1981 was stamped "Payment Received" of the became effective for non-payment of premium. Payment was in
amoung of P930.60 on "12-24-81" by Domingo Adora. 14 This is fact made, rendering the policy operative as of June 22, 1981, and
removing it from the provisions of Article 77, Thereafter, the policy There is no proof that the notice, assuming it complied with the
could be cancelled on any of the supervening grounds enumerated other requisites mentioned above, was actually mailed to and
in Article 64 (except "nonpayment of premium") provided the received by Pinca. All MICO's offers to show that the cancellation
cancellation was made in accordance therewith and with Article 65. was communicated to the insured is its employee's testimony that
the said cancellation was sent "by mail through our mailing
A valid cancellation must, therefore, require concurrence of the section." without more.
following conditions:
On the other hand, there is the flat denial of Pinca, who says she
(1) There must be prior notice of cancellation to the insured; never received the claimed cancellation

(2) The notice must be based on the occurrence, after the effective It stands to reason that if Pinca had really received the said notice,
date of the policy, of one or more of the grounds mentioned; she would not have made payment on the original policy on
December 24, 1981. Instead, she would have asked for a new
(3) The notice must be (a) in writing, (b) mailed, or delivered to the insurance, effective on that date and until one year later, and so
named insured, (c) at the address shown in the policy; taken advantage of the extended period. The Court finds that if she
did pay on that date, it was because she honestly believed that the
policy issued on June 7, 1981, was still in effect and she was willing
(4) It must state (a) which of the grounds mentioned in Section 64 to make her payment retroact to July 22, 1981, its stipulated
is relied upon and (b) that upon written request of the insured, the commencement date.
insurer will furnish the facts on which the cancellation is based.

You might also like